NURX108 Transition to the Professional Nurse Role

¡Supera tus tareas y exámenes ahora con Quizwiz!

Management of Patient Centered Care What aspects would the principled, critically-thinking RN manager consider when making decisions about delegation of resources? (Select all that apply.) The correct task To the correct person Under any circumstances Under accurate evaluation With accurate communication

*1) "The RN uses critical thinking and professional judgment when following the Five Rights of Delegation, to be sure that the delegation or assignment is: 1. The right task 2. Under the right circumstance 3. To the right person 4. With the right directions and communications; and 5. Under the right supervision and evaluation." *2) See 1). 3) It should be "under the right circumstances," not any circumstances. *4) See 1). *5) See 1).

Management of Patient Centered Care A newly licensed RN is competent in which activity? Titrating the dose of chemotherapy Accessing an implanted venous access device Assisting the physician in surgery Creating a patient-specific plan of care

1) Administering chemotherapy requires additional education, and is not an expectation of a new RN. 2) New RNs require time to develop technical competency in various skills; however, knowledge of associated principles, such as aseptic technique, is expected. 3) RNs or surgical technicians may assist in surgery; however, this activity requires specialized education not included in pre-licensure nursing programs. *4) Planning care for patients is a core competency for the registered nurse.

Professional Role Development Which action by an RN demonstrates a professional quality? Practicing as a graduate nurse prior to taking the NCLEX-RN exam Analyzing patient data obtained by unlicensed assistive personal Deferring all patient care decisions to a health care provider Using past tradition as the foundation for nursing practice

1) Professional service entails ethical commitment and legal responsibility to the public. Within this service, the public must be guaranteed the competence of professional RNs by licensure. *2) Many values are important to the growth and development of a professional, just as characteristics of commitment, accountability, responsibility, ethical and moral standards, and caring are important to the maturation of the profession as a whole. 3) By definition, autonomy means that the individual has self-determination over functions within the workplace. A professional must be responsible and accountable for her or his actions. RNs have had difficulty achieving autonomy, because of the hierarchal nature of nursing organizations and the continued dependence on the medical profession. 4) Professional practice is based on a body of knowledge gained from research and experience. The education is a combination of general and specialized courses.

Professional Role Development What should the RN do to become certified by the American Nurses Credentialing Center (ANCC) in surgical nursing? Go back to school and obtain a master's degree. Demonstrate current practice and knowledge by passing an approved exam. Acquire practical knowledge and experience by attending a six-week course. Apply for recognition as an expert.

1) See 2). *2) ANCC certification is a method of recognizing nurses who have special expertise. Applicants must demonstrate current practice and knowledge beyond that required for licensure as an RN. After submitting evidence of completing all requirements, an RN may take the national examination for a specific certification. 3) See 2). 4) See 2).

Ethical, Legal, Regulatory, Professional Implications Which statement best describes the purpose of nurse practice acts? They establish minimum fees to be charged for services rendered by licensed nurses. They protect the public and exert legal control over nursing practice. They detail methods for validating the competency of nurses in specialty areas of practice. They specify criteria for accrediting institutions and registering educational programs.

1) See 2). *2) Nurse practice acts are laws established in each state to protect the public and regulate the practice of nursing. 3) See 2). 4) See 2).

Professional Role Development Which action is included in the National League for Nursing (NLN) Competencies for the Associate Degree Graduate? Disseminate research findings. Make nursing judgments. Create systems to promote human flourishing. Provide leadership in health reform.

1) The competencies of the associate degree graduate include human flourishing, nursing judgment, professional identity, spirit of inquiry. Disseminating research finding is a competency at the doctorate level. *2) Nursing judgment is one of the four competencies the NLN expects of AD graduates. 3) Creating systems to promote human flourishing is a competency of the master's prepared RN. 4) Demonstrating leadership in health care reform is a competency at the doctorate level prepared RN.

Management of Patient Centered Care Which action would an RN nurse manager encourage nursing team members to take prior to providing culturally proficient nursing care? Reflect on their own personal beliefs as compared to the beliefs of others. Review institutional policies that promote cultural awareness. Understand more than one language for effective communication. Assign nursing staff to patients of matching cultural backgrounds.

*1) A culturally proficient and competent RN is aware of one's own beliefs as well as the beliefs of others, and how the two may be and often are different. 2) Policies to promote cultural awareness helps toward becoming culturally proficient; however, teaching others how to put the policy into practice through self-reflection would be a better option. 3) Although understanding different languages will help with communication, it does not make the RN automatically culturally proficient. 4) Assigning nursing staff to patients of like cultures does not render the rest of the staff to be culturally competent.

Ethical, Legal, Regulatory, Professional Implications Which is the most important resource he RN must research before seeking employment as an RN in another state? Nurse practice act State board of nursing American Nurses Association (ANA) Code of Ethics National League for Nursing (NLN) Position Statements

*1) A nurse practice act is state legislation regulating the practice of nurses to protect the public, define the scope of practice, and make nurses accountable for their actions. Nurse practice acts establish state boards of nursing (SBNs) and define specific SBN powers regarding the practice of nursing within each state. Rules and regulations written by the SBN become statutory laws under powers delegated by the state legislature. 2) The state board of nursing is a regulatory body, not a resource for research. 3) Although this is important information to know, it does not change based on which state an RN is practicing in. 4) Although this is important information to know, it does not change based on which state an RN is practicing in.

Management of Patient Centered Care A patient with chronic obstructive pulmonary disease (COPD) is being discharged and requires teaching regarding activity restrictions. To which interprofessional team member should this task be assigned? Registered nurse (RN) Physical therapist (PT) Unlicensed assistive personnel (UAP) Licensed practical nurse/Licensed vocational nurse (LPN/LVN)

*1) A registered nurse (RN) is the only interprofessional team member who can provide discharge teaching to patients. 2) A physical therapist may be consulted for mobility concerns but the teaching requires information about restriction of activity which is not within that team member's responsibility. 3) Unlicensed assistive personnel (UAP) cannot provide discharge teaching to patients. 4) A licensed practical nurse/Licensed vocational nurse (LPN/LVN) cannot provide discharge teaching to patients.

Professional Role Development What are educational outcomes identified by the National League for Nursing (NLN) for the Associated Degree Nurse (ADN) graduate? (Select all that apply.) Exhibit professional behaviors. Implement clinical decision making. Demonstrate caring behaviors. Determine effective resource utilization. Design research studies.

*1) According to the National League for Nursing, Council of Associate Degree Nursing (2000a), Educational competencies for graduates of associate degree nursing program, the eight core competencies of graduate entry-level RN include professional behavior, communication, assessment, clinical decision making, caring intervention, teaching and learning, collaboration, and managing care. *2) See 1). *3) See 1). 4) According to the American Nurses Association (ANA) Nursing's social policy statement, resource utilization is one of the nine standards of professional practice. 5) According to the American Nurses Association (ANA) Nursing's social policy statement research design requires further educational preparation.

Ethical, Legal, Regulatory, Professional Implications Nurse practitioners exercising their authority to prescribe appropriate medications would be an example of which recommendation outlined in the Institute of Medicine (IOM) Report on the Future of Nursing? RNs should practice to the full extent of their education and training. RNs should join collective bargaining efforts to improve working conditions. RNs should be full partners with other health care professionals in redesigning health care in the US. RNs should achieve higher levels of education and training through an improved education system that promotes seamless academic progression.

*1) An RN practitioner who utilizes prescriptive authority is an example of a RN who is working to the fullest extent of their education and training. 2) Participation in collective bargaining is not a recommendation of the IOM Report on the Future of Nursing. 3) The IOM recommends that RNs partner with health care providers and health care Professionals; however, this is not related to prescriptive authority. 4) The IOM recommends that RNs achieve higher levels of education and training through seamless educational progression; however, this is not related to prescriptive authority.

Management of Patient Centered Care Which actions demonstrate an RN's awareness of the value and relevance of research in nursing? (Select all that apply.) Learning to critically scrutinize the status quo Reading clinical journals regularly but also critically Seeking work environments that promote the use of research findings and evidence-based care Working alone to apply established findings of nursing and other health-related research to practice Learning to look for evidence that clearly supports the effectiveness and feasibility of updating nursing interventions

*1) An associate degree RN would demonstrate awareness of the value and relevance of research in nursing by performing the following activities that are within their scope of practice: learning to critically scrutinize the status quo; reading clinical journals regularly but also critically; seeking work environments that promote the use of research findings and evidence-based care; learning to look for evidence that clearly supports the effectiveness and feasibility of updating nursing interventions. An associate degree nurse's scope of practice pertaining to research includes demonstrating the value and relevance of research in nursing; assisting with identifying problem areas in nursing practice; and assisting in collection of data within an established structured format. *2) See 1). *3) See 1). 4) An associate degree nurse would not work alone to apply established findings of nursing and other health-related research into practice. *5) See 1).

Management of Patient Centered Care An RN in the emergency department (ED) employs an authoritarian style of leadership. Which actions demonstrated by the RN align with this type of leadership style? (Select all that apply.) Suppresses creativity of group members Values collaboration and autonomy Provides directions by giving orders Exerts a high degree of control Possesses strong interpersonal skills

*1) An authoritative leader is one who makes the decision and does not seek input from others. 2) Collaboration and autonomy is seen in the democratic style of leadership not the authoritarian type. *3) Giving orders is characteristic of an authoritarian leader and can be useful in emergency situations. *4) Authoritarian leaders tend to control the situation and take on the responsibility of making the final decisions. 5) Authoritarian leaders do not posses strong interpersonal skills. They are more concerned about meeting goals.

Management of Patient Centered Care An RN directs the nursing team using clear and concise commands during a patient emergency. Which style of leadership is in the RN using? Autocratic Democratic Laissez-faire Transformational

*1) An autocratic style of leadership (also called authoritarian) maintains strong control over all aspects of the group and its activities. 2) In the democratic style of leadership, all aspects of the process of achieving a goal, from planning and goal setting to implementing and taking credit for the success of the project is shared by the group. 3) In the laissez-faire style of leadership the group members are allowed to determine their own goals and the methods to achieve them. 4) Transformational style leadership is another name for democratic style of leadership.

Management of Patient Centered Care A patient gets out of bed to go to the bathroom one night and falls. Using a quality improvement process, the RN defines the problem and collects data. What step should the RN initiate next, to complete the process of resolving the cause of the fall? Complete a comprehensive analysis of all factors. Review circumstances surrounding the event. Implement strategies to prevent falls. Review individual patients' risks for falls.

*1) Analysis follows identification of the problem and data collection. 2) This is data collection. 3) This is implementation of improvement strategies. 4) This is data collection.

Which statement reflects the concept of clinical competency for Registered Nurses? 1) Demonstrating clinical skills and problem solving while implementing policies and procedures. 2) Providing patient-centered care to meet identified needs. 3) Applying the nursing process for a variety of patients in clinical settings. 4) Researching evidence-based findings for application to patient care.

*1) Clinical competency is a term that describes performance expectations in clinical practice. 2) Patient-centered care focuses on unique needs of patients, rather than a nurse's performance. 3) The nursing process is a problem solving approach that may assist with demonstrating competence, but is not a comprehensive reflection of clinical competence. 4) Evidence-based findings inform patient care provided by Registered Nurses.

Management of Patient Centered Care An RN is insightfully observing, interviewing, and examining a patient, to gain comprehensive data and information about the patient's condition. Which action is the RN performing? Assessing the patient Identifying clinical issues Establishing priorities of care Determining a plan of action

*1) During the assessment phase of the nursing process an RN establishes a comprehensive data through the observation, interviewing, and examining of a patient to gain insight and information about the patient's condition or situation. 2) Identifying clinical issues follows collecting data. 3) Determining a plan of action follows analyzing the data collected. 4) See 1).

Management of Patient Centered Care Which statement by the RN demonstrates the use of evidence in nursing practice? (Select all that apply.) "We incorporate the best available research with patient preferences when providing care." "We use experiential learning to develop nursing knowledge and skills." "We have implemented research protocols without concern of financial cost." "We use statistical data on the risk-benefit ratio before making health care decisions." "We have patient care interventions based on both science and caring."

*1) Evidence-based practice emphasizes improving quality of care for individuals while improving health care outcomes. Coordinating and balancing patient preferences with best available research is the standard for nursing practice. 2) Experiential learning is also called the trial-and-error process, and it is not based on the best evidence or research, if any at all. 3) Evidence-based practice decreases cost of operations while improving quality of care. 4) The medical model of health care uses statistical data to estimate the risk-benefit ratio. However, the decisions made need to incorporate patient preference. *5) Nursing is a profession that uses scientific inquiry and caring in daily patient practice.

Ethical, Legal, Regulatory, Professional Implications What is the main purpose of nursing licensure? To demonstrate that the RN can function at a minimal level of competency To identify professionals at an advanced level of competency To validate that mandatory hours of instruction have been met To provide a mechanism for tracking the number of nurses in the profession

*1) For any professional dealing with the public, it is necessary to demonstrate ability to function at lease a minimal level of competency. 2) Licensure deals with minimal competency, not advanced competency. 3) Licensure may require mandatory hours, but the main purpose is to ensure competency, not number of hours of instruction. 4) Licensure may provide data on the number of nurses, but that is not its main purpose.

Management of Patient Centered Care RNs play an important role in quality improvement by advancing the national initiative of Healthy People 2020. Which nursing competency is key to achieving the goal of quality improvement as set forth in Healthy People 2020? Promoting healthy behaviors through patient teaching Using evidence-based practices when implementing care in hospitals Implementing protocols for a culture of safety in acute care settings Participating in interdisciplinary teams focused on quality improvement

*1) Goals of Healthy People 2020 are preventing disease and improving health of all groups, making patient teaching of healthy behaviors the most important nursing competency to address these goals. 2) Evidenced-based practices are essential to providing high quality care but do not relate to prevention. 3) Protocols for a culture of safety focus on reducing risks that result from care rather than focusing on promoting health. 4) The quality improvement focus of interdisciplinary teams is not a goal of Healthy People 2020.

Professional Role Development In caring for an increasingly diverse geriatric population, which action is necessary for an RN to take, so each patient remains a central partner in health care? Tailor patient teaching to meet the needs of multiple cultures. Standardize safety measures to ensure all patients remain safe. Educate patients to conform to the hospital environment for safety. Use a family member for translation when teaching a patient, if necessary.

*1) Hospitals today offer teaching materials and a medical translator service in multiple languages if the cultural issue is related to a language difference. Understanding common cultural differences and issues will also help the RN teach the patient more effectively. 2) This is not the best option because standardizing care for all patients does not account for the specialized needs of the geriatric population or culturally specific safety considerations. 3) The RN should understand that a patient's culture may require alternative considerations, and that the RN may need to have an understanding of that particular culture. Expecting patients to adhere to hospital safety standards only will put the patient at risk. 4) A family member is not the standard for translating information with the patient. The patient may withhold information, because of privacy reasons. Using a facility's medical translator service is the standard of care when rendering care for patients who require translation.

Professional Role Development An RN is collaborating with a patient's health care provider, physical therapist, and occupational therapist to develop a plan of care. This is an example of which Institute of Medicine (IOM) competency? Interdisciplinary teamwork Evidence-based practice Quality improvement Cost-effective care

*1) Interdisciplinary teamwork is the coordination of care with other care providers. 2) Evidence-based practice includes clinical expertise, patient values, and research in the decision making process. It does not necessarily involve collaboration with other care providers. 3) Quality improvement measures and addresses issues in health care quality. It does not necessarily involve collaboration with other care providers. 4) Working to make care more cost-effective does not necessarily involve collaboration with other care providers.

Management of Patient Centered Care An RN is preparing to perform discharge teaching for a patient who has a limited understanding of English. To ensure understanding of the instructions, what are appropriate actions the RN could take when providing teaching? (Select all that apply.) Provide the instructions verbally and in writing. Use simple language and familiar terms. Provide the instructions in the patient's native language. Give the instructions to the patient alone, to maintain privacy. Ask the patient's children to assist with translating medical terms.

*1) It is best to provide instructions in multiple ways to promote a more thorough comprehension and understanding of the material. Providing written instructions to compliment verbal instructions allows the patient to refer to instructions once they have left the facility if things have been forgotten or are unclear. *2) Simplify terms and avoid jargon when language and cultural background varies. *3) Instructions and education should always be delivered in a patient's native language (language they understand best) to promote understanding of instructions. 4) Family members such as spouse's are resources to increase understanding and adherence to regime. 5) Children of the patient may not be qualified to interpret health care information.

Ethical, Legal, Regulatory, Professional Implications An RN speaking about health care reform states, "Health care is a right, but with every right comes responsibility, to engage in healthy lifestyles." Which primary ethical principle is addressed with this position statement? Justice Fidelity Autonomy Beneficence

*1) Justice is the obligation to be fair to all people in the distribution of benefits and burdens. Rights are benefits and responsibilities can be seen as burdens. The statement addresses the question of whether it is fair for costs of health care to be equally divided among both those who accept the responsibility for healthy living and those who do not and hence set themselves up for needing to use more resources. 2) Fidelity means to be faithful to one's agreements or responsibilities. The statement suggests that financial responsibilities for health care be proportioned based on risk factors; it is not addressing fulfillment of responsibilities. 3) Autonomy refers to self-rule and the ability to make independent personal decisions. This is not an unqualified right; the right to self-determination is lost when endangerment or harm to others would result. The statement is not directly addressing the right to choice; it is addressing the consequences of choice. 4) Nonmaleficence is the obligation to do no harm - intentionally, unintentionally or by placing a person at risk. It could be said that putting additional financial burden on those with an unhealthy lifestyle is creating a risk for harm but the converse is also true, increasing cost on those with healthy lifestyles to cover the cost of care for those with unhealthy lifestyles can be putting them at risk. In any case, this is not the primary issue in the case.

Management of Patient Centered Care What is one task that the RN can delegate to the LPN/LVN in the acute care setting? Administering an enteral feeding Completing an admission assessment Revising a patient's plan of care Setting short- and long-term patient goals

*1) LPN/LVNs can provide basic therapeutic nursing skills such as feeding. 2) LPN/LVNs can collect data, but the RN performs the extensive admission assessment to identify the problems. 3) The RN evaluates the effectiveness of the overall plan, and modifies and revises the plan as needed, not the LVN/LPN. 4) The RN, along with the patient, sets the short- and long-term patient goals.

Professional Role Development Which term was added to most state nurse practice acts to reflect the expansion of the RN's role into the statutory realm? Nursing diagnosis Nursing assessment Nursing plan of care Nursing standards of care

*1) Many state legislatures have responded to the expanded role of the nurse by broadening the scope of their nurse practice acts. For example, adding the term "nursing diagnosis" to many states' nurse practice acts reflects the legislature's recognition of the expansion of the nurse's role into the statutory realm. 2) See 1). 3) See 1). 4) See 1).

Ethical, Legal, Regulatory, Professional Implications Which is a common reason for nurses to leave their staff positions? Moral distress felt by RNs providing patient care. Unrealistic expectations by patients who demand care. Cused by health care reform. Poor communication between nursing staff and hospital management.

*1) Moral distress by RNs feeling they are unable to provide adequate or safe care is a common cause of RNs leaving a job or even the profession. 2) The issue of inadequate staffing is not caused by unrealistic patients. 3) The larger issue of health care reform rarely influences nursing attrition. 4) There is no evidence to support poor communication as reason nurses are leaving the hospital.

Ethical, Legal, Regulatory, Professional Implications Which document defines the legal scope of practice for RNs? State Legislatures' Nurse Practice Acts The Joint Commission Standards American Nurses Association (ANA) Code of Ethics for Nurses with Interpretative Statements American Nurses Association (ANA) Standards of Professional Nursing Practice

*1) Nurse practice acts evolved as licensure laws were enacted. Their purpose is to define nursing, stipulate the qualifications to practice nursing, outline the methods of obtaining licensure, licensure renewal, and interstate endorsement or reciprocity, establish and maintain rules and regulations of nursing, and delineate unlawful acts, misconduct, or disciplinary actions. 2) Legal scope of practice is defined by governments, not professional organizations. 3) Legal scope of practice is defined by governments, not professional organizations. 4) Legal scope of practice is defined by governments, not professional organizations.

Management of Patient Centered Care Which are characteristics of a blame-free, just culture? (Select all that apply.) Committing to transparency Using errors as learning opportunities Holding staff accountable for at-risk behaviors Analyzing events to identify system improvements Reassigning rather than dismissing a person who made an error

*1) Nursing cultures need to make quality and safety a priority rather than focusing on what went wrong and blaming someone for the error. Errors should not be covered up but identified and addressed. *2) Nursing cultures need to make quality and safety a priority rather than focusing on what went wrong and blaming someone for the error. Errors should be used as opportunity to learn and improve the system. *3) Nursing cultures need to make quality and safety a priority rather than focusing on what went wrong but does not tolerate subpar behavior. Staff must be held accountable for at-risk or reckless actions but at the same time be prepared to handle human error occurrences. *4) Nursing cultures need to make quality and safety a priority rather than focusing on what went wrong and blaming someone for the error. Analyzing an event can positively impact the future quality of care and safety. 5) Reassigning the person who made the error can be seen as punitive and might cause someone to try and cover up the mistake.

Ethical, Legal, Regulatory, Professional Implications According to research, which characteristics set magnet-designated hospitals apart from other hospitals? (Select all that apply.) Decreased mortality rates Increased patient satisfaction New innovations and improvements Exemplary professional performance Improved patient safety and quality care

*1) Research related to magnet designated hospitals has documented an overallimprovement in patient safety and quality of care. Specific research findings include decreased mortality rates. *2) Research related to magnet designated hospitals has documented an overall improvement in patient safety and quality of care. Specific research findings include increased patient satisfaction. 3) In 1983, the American Academy of Nursing (AAN) Task Force on Nursing Practices studied work environments in hospitals that attracted and retained well-qualified nurses who promoted quality patient care. Hospitals that demonstrated these qualities were designated as "magnet hospitals". Building upon this study, the American Nurses Association (ANA) board of directors approved a proposal for the Magnet Hospital Recognition Program for Excellence in Nursing Services. The most current Magnet Recognition Model Program has 5 components: transformational leadership; structural empowerment; exemplary professional practice; new knowledge, innovation, and improvements; and empirical quality results. 4) See 3). *5) Research related to magnet designated hospitals has documented an overall improvement in patient safety and quality of care. Specific research findings include fewer patient falls and a decrease in the number of pressure ulcers.

Professional Role Development A novice RN reviews the policy and procedure manual on the nursing unit before performing a procedure on a patient. Which primary role socialization has the novice RN achieved? Taking on the values of the profession Modifying the professional role to one that is personally acceptable Acquiring technical and theoretical skills for professional development Balancing the professional role with the personal role

*1) Reviewing procedures beforehand reflects that the RN has been socialized into the values of the profession. 2) Reviewing procedures beforehand does not reflect modification of the professional role for personal gain. 3) Reviewing procedures beforehand may give the RN new technical expertise for professional development, but it does not reflect socialization into the primary role of upholding the value of patient-centered care. 4) Reviewing procedures before implementing them shows the novice RN is putting the professional value of caring before the personal.

Professional Role Development RNs have the capacity to promote healthy habits using which methods? (Select all that apply.) Modeling wellness and prevention Practicing self-care as a life-long habit Enjoying self-indulgence on an occasional basis Choosing behaviors that reduce stress Choosing behaviors that balance stress

*1) Richards (2013) wants nurses to lead by example, "creating healthy habits for ourselves, flourishing as ambassadors of self care." *2) This is an example of modeling healthy habits. 3) Self-care is not self-indulgence, which is temporary and often a symbolic fix. *4) This is an example of modeling healthy habits. *5) This is an example of modeling healthy habits.

Management of Patient Centered Care What is the policy of the Centers for Medicare and Medicaid Services (CMS) Medicare program regarding so-called "never" events, where a reasonably preventable medical error has occurred? Hospitals will not be reimbursed for costs of never events. Hospitals will be fined based on the extent of harm caused by the never event. Hospitals will lose accreditation by The Joint Commission if a never event occurs. Hospitals will be required to implement actions to prevent never events before Medicare patients can be admitted.

*1) Starting in 2007, the CMS changed its Medicare payment policy so that they would no longer have to pay for reasonably preventable medical errors that occur in the hospital. 2) CMS withholds payment so the hospitals have to cover Medicare costs of never events, but no fines are imposed on the hospital. 3) Accreditation by The Joint Commission is not determined by CMS policies. 4) Hospitals cannot limit admission of Medicare patients.

Ethical, Legal, Regulatory, Professional Implications Which action indicates emotionally intelligent RN leadership? The RN leader takes time to reflect prior to responding to a clinical issue. The RN leader maintains a strict and firm lateness policy with no exceptions. The RN leader holds all employees to the same high standards the RN leader holds. The RN leader is successful at achieving the desired outcome no matter what the cost.

*1) Taking time to pause before responding is a sign of an emotionally intelligent leader. 2) RNs with high emotional intelligence demonstrate high levels of empathy. 3) All employees are not the same as the RN leader. RN leaders with high emotional intelligence demonstrate high levels of empathy for each individual. 4) Achieving the desired outcome no matter what the cost is a sign of low emotional intelligence.

Management of Patient Centered Care Which are incorporated in the guidelines under The Joint Commission's "Leadership" standard that deal with lateral violence? (Select all that apply.) Define behaviors that are considered disruptive in a code of conduct. Implement a process for managing individuals who are displaying disruptive behaviors. Require staff to demonstrate interpersonal skills and interprofessionalism in the credentialing process. Implement zero tolerance policies for disruptive behaviors. Provide support to victims of disruptive behavior.

*1) The Joint Commission (TJC) developed new guidelines under their "Leadership" standard to deal with behaviors that are interpreted as lateral violence. These include defining behaviors that are considered lateral violence. *2) The Joint Commission (TJC) developed new guidelines under their "Leadership" standard to deal with behaviors that are interpreted as lateral violence. These include requiring hospital administration to develop and implement a process for managing individuals who are displaying disruptive and inappropriate behaviors. *3) The Joint Commission (TJC) developed new guidelines under their "Leadership" standard to deal with behaviors that are interpreted as lateral violence. These include requiring additional standards for medical staff to follow for the credentialing process, including demonstrating interpersonal skills and recognizing interprofessionalism. 4) This is not addressed in the TJC guidelines. 5) This is not addressed in the TJC guidelines.

Professional Role Development Which organization is responsible for the development and validation of the licensing examinations for registered and practical nurses? National Council of State Boards of Nursing Commission on Collegiate Nursing Education American Nurses Credentialing Center American Association of Colleges of Nursing

*1) The National Council of State Boards of Nursing is responsible for the development and validation of the licensing examination for registered and practical nurses. 2) The Commission on Collegiate Nursing Education - accredit nursing education programs. 3) The American Nurses Credentialing Center - operates the magnet hospital recognition program. 4) The American Association of Colleges of Nursing - focuses on quality of education in baccalaureate and higher degree nursing programs.

A team of RNs on a medical nursing unit developed and implemented a new system to improve hand-off reports. One RN, who returned from an extended leave to find the new system already in place, refuses to use it. Which is the most likely reason for this RNs resistance to change? 1) Lack of involvement in the decision making process 2) Lack of research that supports the need for change 3) Increase in the time required for documentation 4) Lack of familiarity with the new process

*1) The RN is resistant to change because the RN was not available to be actively involved in the change process. 2) Resisting change is not related to lack of research done to validate the need for change. 3) There is no data that the change increases documentation time. 4) Lack of familiarity does create some resistance however the lack of involvement in decision process is a major factor in resistance to change process.

Ethical, Legal, Regulatory, Professional Implications What purpose does the American Nurses Association (ANA) Code of Ethics serve in representing the behaviors and beliefs underlying the foundation of nursing practice? It is a statement of moral obligations required of RNs. It outlines possible responses to particular ethical challenges. It informs patients of the risks, benefits, and outcomes of nursing care. It is designed to differentiate RNs from other health care professionals.

*1) The code provides a basis for ethical analysis and decision making and establishes the ethical standard for the profession. 2) The wording is intentionally vague so the code can serve as general guidelines. 3) The code is directed to guide RNs in their practice and not inform patients about nursing care. 4) The code does not mention any differentiation between RNs and other health professions.

A health care provider has discussed potential treatment options for a patient in a coma with the patient's family. One of the family members asks the RN, "What would you do if it was your loved one?" What is the most appropriate response the RN can make in this situation? 1) "What do you think the patient would want done?" 2) "It sounds like you're unsure which the treatment options to choose." 3) "I've seen patients live many years with the treatment." 4) "Would you like the provider to explain the options to you again?"

*1) The designated decision maker is encouraged to make decisions as the patient would have made, if able to do so. 2) Reflecting back that the family member is having a difficult time making a choice is not helpful in helping the family arrive at a decision. 3) Providing personal opinion or observation does not help the family member to make a decision that the patient may have made, if the patient was able to do so. 4) There is no evidence that the family member did not hear the options as outlined by the health care provider.

Ethical, Legal, Regulatory, Professional Implications What is an expectation of an RN who is a responsible member of an Ethics Committee? (Select all that apply.) Serve as a resource to staff in resolving ethical dilemmas. Resolve negligence or malpractice claims by patients or families. Provide educational programs on ethical principles to hospital staff. Establish hospital policies on the use of medical procedures involving technology. Evaluate individual staff members to see if they follow ethical and legal standards.

*1) The primary purpose of an Ethics Committee is to help in the resolution of complex ethical dilemmas. 2) Negligence and malpractice are legal rather than ethical issues. *3) Providing educational programs on ethical principles for staff members is a common role of an Ethics Committee. 4) The Ethics Committees can evaluate ethical implications of policies but would not establish medical procedures. 5) Evaluating individual staff is a management responsibility and is not appropriate for an Ethics Committee.

Professional Role Development What is the primary purpose of the National League for Nursing (NLN)? To maintain and improve the standards of nursing education To work internationally to improve the quality of care globally To recognize individuals' contributions of leadership and service to the profession of nursing To provide a political voice for the improvement in access to care for the homeless

*1) The primary purpose of the National League for Nursing is to maintain and improve the standards of nursing education. 2) Global improvement in health and nursing care is the goal of the International Council of Nursing and not the NLN. 3) Sigma Theta Tau is a professional honors organization that recognizes contributions in service and leadership of nurses - this is not the purpose of the NLN. 4) The American Nurses Association (ANA) includes as one of its purposes the improvement of standards of health and access to health care services for everyone. This is not the primary purpose of the NLN.

Ethical, Legal, Regulatory, Professional Implications What is the purpose of state nurse practice acts and subsequent policies for implementing and enforcing laws? To provide consumer protection To recognize barriers to effective delegation To differentiate among types of patient care plans To identify strategies to make patient care available

*1) The role of the nurse practice acts, and policies created by the state boards of nursing for implementing and enforcing laws using those acts as a legal basis, is to protect the consumer. 2) See 1). 3) See 1). 4) See 1).

Ethical, Legal, Regulatory, Professional Implications Each state board of nursing has which types of authority in common? (Select all that apply.) Granting licensure Approving nursing programs Deciding policies on nursing procedures Regulating safety in health care facilities Establishing standards for nursing schools

*1) The state boards of nursing (SBNs) are considered regulatory powers because they provide the SBN with control over nurses according to rule, principle, or law. By legislative act, all SBNs have the power to grant licensure, approve nursing programs, establish standards for nursing schools, and write specific regulations for nurses and nursing practice in general in that state. *2) See 1). 3) The health care facility or employer develops policies on nursing procedures. 4) The state boards of nursing (SBNs) do not regulate safety in health care settings. *5) See 1).

Ethical, Legal, Regulatory, Professional Implications An RN is describing the role of the nurse coder to a group of high school students at a career fair and notes rapid growth in the field of nurse coding. What primary reasons would the RN cite for this growth? (Select all that apply.) Aging of the population Advances in scientific research Coming introduction of ICD-10 coding system Increased use of advanced electronic technology Escalating development of new diagnostic techniques

*1) The three primary reasons for growth in the nurse coder role are the coming change from ICD-9 to ICD-10, the aging of the population and increased use of advanced electronic technology. An aging population is a primary factor because of related increases in need for health care and by extension, an increased need for coding since reimbursement is contingent on that coding. 2) See 1), 3), and 4). *3) The three primary reasons for growth in the nurse coder role are the coming change from ICD-9 to ICD-10, aging of the population and increased use of advanced electronic technology. ICD-9 has 14,000 codes and ICD-10 has more than 70,000. The increase in codes will increase coding time and hence the need for coders. *4) The three primary reasons for growth in the nurse coder role are the coming change from ICD-9 to ICD-10, aging of the population and increased use of advanced electronic technology. The increased use of advanced electronic technology makes more specific coding possible and hence, makes the coming of ICD-10 with its more than 70,000 codes possible. An increase in codes requires increased time for coding and hence increases the need for coders. 5) See 1), 3), and 4).

Management of Patient Centered Care Which exemplifies the RN using an accommodation method towards conflict resolution? An RN performs all the work that would normally be delegated. An RN remains neutral when witnessing an argument between two other RNs. An RN asserts a strong desire to win a conflict by out-talking other participants An RN considers all aspects of the conflict and creates a solution that is agreeable to all participants involved.

*1) This action reflects an accommodation method of conflict resolution. 2) This action reflects an avoidance method of conflict resolution. 3) This action reflects a competition method of conflict resolution. 4) This action reflects a compromise method of conflict resolution.

Ethical, Legal, Regulatory, Professional Implications What are some reasons that a state board of nursing would revoke a nursing license? (Select all that apply.) Use of illegal drugs while providing patient care Demonstration of negligence in the practice of nursing Conviction of a misdemeanor while continuing to practice nursing Failure to renew a nursing license while continuing to practice nursing Failure to provide a patient's list of prescribed medications to an employer

*1) This is a reason of state board of nursing would revoke a nursing license. *2) This is a reason of state board of nursing would revoke a nursing license. 3) Conviction of a misdemeanor is not grounds for revocation of a nursing license. *4) This is a reason of state board of nursing would revoke a nursing license. 5) In general an RN should not be providing such a list, so failure to do so is not grounds for revocation of a nursing license.

Ethical, Legal, Regulatory, Professional Implications Which data points support a health care reform position that, compared to many other countries, significantly more money is spent per capita on health care in the US, but the spending does not result in better outcomes? (Select all that apply.) The US infant mortality rate ranked 23rd out of 23 wealthy countries. The US ties for the lowest healthy life expectancy among 23 wealthy countries. People in the US are more likely to receive reminders about preventive care than people in other countries. The proportion of people seeking basic primary care in hospitals is higher in the US than in other countries. The amount of money spent on medical research is higher in the US than in other countries.

*1) This is a true statement about a key measure of health outcomes which illustrates that despite the large amount of money spent on health care in the US, health outcomes are not as good as those in countries which spend less on health care. *2) This is a true statement about a key measure of health outcomes which illustrates that despite the large amount of money spent on health care in the US, health outcomes are not as good as those in countries which spend less on health care. 3) Money spent on reminding people about preventive care is not likely to worsen outcomes. *4) This is a true statement explaining how there might be a link between high costs and poor outcomes, since people receiving primary care in hospitals are typically foregoing preventive care and engaging with health care providers when conditions are more expensive to treat. 5) This is a true statement, but it argues against the position, since it explains and justifies why the direct link of spending to outcomes might not be legitimate.

Management of Patient Centered Care Which tasks are appropriate for unlicensed assistive personnel (UAP) to perform? (Select all that apply.) Ambulation Basic hygiene Review of incentive spirometry Admission assessment Administration of oral medications

*1) This is an appropriate task for a unlicensed assistive personnel (UAP) to perform. *2) This is an appropriate task for a unlicensed assistive personnel (UAP) to perform. 3) This task would be performed by a licensed practical nurse (LPN). 4) This task would be performed by a registered nurse (RN). 5) This task would be performed by a licensed practical nurse (LPN).

Professional Role Development After attending a nursing class on death and dying, a nursing student asks the question, "How might someone experience death whose cultural background is different from mine?" Which domain of learning is the student developing? Affective Cognitive Kinesthetic Psychomotor

*1) This type of question is addressing values and feelings. 2) This would include questions such as "How does this relate to last week's learning?" 3) This is a learning style, not a learning domain. Kinesthetic style of learning deals with feeling, touching, and manipulating, such as redressing a wound. 4) This would include questions such as "What are the principles underlying this procedure?"

Which leadership actions assist nurse managers in facing challenges such as sufficient staffing and ensuring that safe, quality patient care is provided? (Select all that apply.) 1) Establish common goals that can be achieved. 2) Use written warnings to ensure that work is accomplished. 3) Recognize positive patient outcomes and accomplishments by staff. 4) Reschedule staff meetings when busy with other responsibilities. 5) Volunteer to document minutes of meetings so all staff is informed.

*1) To lead and make change, nurse managers must recognize need to have staff involved. 2) The use of threats to manipulate staff to be effective members is ineffective and counterproductive. *3) Recognition of work that is well done reinforces desirable behavior. 4) Frequent rescheduling of meetings may be perceived by staff that their contributions are less important than nurse manager's time. 5) Record keeping responsibilities should be delegated to a staff member so nurse manager can devote time to responsibilities that can only be carried out by manager.

Ethical, Legal, Regulatory, Professional Implications Which statement regarding licensure revocation is accurate? A patient's family may make a claim that results in revocation. The lack of clear guidelines makes unprofessional conduct difficult to prove. RNs with revoked licenses may retain the right to practice specified nursing duties if they are supervised. The nursing supervisor may revoke licenses to practice in specific facilities.

*1) Unprofessional conduct can be reported by a nursing peer, a supervisor, a patient, or a patient's family. 2) Each respective state's RN practice act provides guidance with regard to the specifics of unprofessional conduct. 3) License revocation is a serious consequence for the RN because it removes the RN's right to practice. 4) If RNs fail to adhere to the standards of safe practice and if they exhibit unprofessional behavior, they can be disciplined by the state nursing licensing board. Nursing supervisors do not have authority to suspend or revoke licenses.

Management of Patient Centered Care Several nursing team members discusses amongst themselves the negative impact that occurs when one co-worker arrives to work 30 minutes late every day. Which aspect of conflict does this situation demonstrate? Unresolved conflict Aggressive response to conflict Emotional contribution to conflict Power conflict

*1) Unresolved conflicts often result in generalized distrust among staff members, as well as gossiping and rumor spreading, backstabbing and lack of cooperation, isolation of certain staff members, and division and polarization of the staff. 2) Aggressive responses to conflict would involve confrontation with the co-worker, not behind-the-back gossip. 3) Although there may be an emotional contribution, the primary factor here is the lack of resolution leading to gossip and backbiting. 4) There is no particular power imbalance in this situation, so a power conflict is not at issue.

Management of Patient Centered Care When is an RN responsible for providing referrals to a patient? When information or intervention needed by the patient is outside the RN's scope of practice When the RN requires assistance analyzing patient data to determine patient strengths and needs When the RN is establishing systematic collection, validation, and communication of patient data When the RN is executing written nursing care plan orders to move the patient toward expected outcomes

*1) When information or intervention needed by the patient is outside the RN's scope of practice the RN would use refer the pateint to a health care professional that can provide the service. 2) These are the tasks associated with the diagnosis phase of the nursing process. 3) These are the tasks associated with the assessment phase of the nursing process. 4) These are the tasks associated with the implementation phase of the nursing process.

Management of Patient Centered Care With growing emphasis on the implementation of evidence-based practice, why has critical and creative thinking, based on the National League for Nursing (NLN) Spirit of Inquiry, become even more important? (Select all that apply.) Nurses must be able to understand the needs of patients for whom they advocate. Nurses must access, understand, evaluate, and disseminate a rapidly expanding body of nursing information. Nurses must be able to distinguish commonalities and inconsistencies regarding the value of the profession. Nurses must be able to surmount the popular power that exerts pressure on the public's ability to make safe choices. Nurses must overlook the values of the organization that employs them to push the boundaries of traditional practice.

*1) With emphasis on the urgency for implementation of evidence-based practice, critical and creative thinking based on the National League for Nursing (NLN) Spirit of Inquiry have become even more important for the following reasons: RNs must access, understand, evaluate, and disseminate a rapidly expanding body of nursing and other health-related information; RNs must be able to recognize commonalities and inconsistencies regarding the value of the profession; RNs must be able to recognize the values of the organization that employ them; RNs must be able to recognize needs of the patients for whom they advocate, and RNs must be able to recognize even the popular culture that exerts pressure on the public's ability to make safe choices. *2) See 1). *3) See 1). 4) An RN must be able to recognize, but cannot overcome, the influences of popular culture and ubiquitously available information in a digital age, that exerts pressure on the public's ability to make safe choices. 5) An RN must be able to recognize the values of the organization that employ them and work within them.

Professional Role Development A nurse mentor notes that a novice RN is unapproachable, silent, and struggling the last few days. Which action would the nurse mentor initially take? Recommend the novice RN be transferred to a different unit. Report the novice RN's attitude and behavior to the supervisor. Take the novice RN aside and tell them to adjust their perspective. Recognize the signs of stress and discuss the problem with the novice RN.

1) A mentor guides and encourages learning and growth in the nursing profession. This action would not give the novice RN a chance to learn and grow in the situation. 2) A mentor guides and supports novice RNs; this action would not be supportive. 3) See 2). *4) Stress can occur when the perceived role of the RN may have to be set aside for the more realistic performed role, from communicator to task organizer. In these instances, the challenge of meeting all of the patient's physical, psychological, social, and spiritual needs becomes less possible, and the care plan becomes more brief.

An RN is assigned to assist a patient who was recently diagnosed with cancer, to help the patient and family deal with complex care issues and barriers related to the treatment plan. What is the term used for this type of nursing practice? 1) Nurse coder 2) Nurse navigator 3) Nurse entrepreneur 4) Patient safety officer

1) A nurse coder reviews documentation to ensure each patient has the appropriate ICD-10 codes for reimbursement issues. *2) A nurse navigator tends to be focused on one specialty area and helps the patient and family move through the treatment plan easier. 3) A nurse entrepreneur is someone who establishes and runs a business. 4) A patient safety officer works to lower the risk factors which cause adverse patient outcomes.

Ethical, Legal, Regulatory, Professional Implications A patient falls after an RN leaves the patient's bed in the high position, with two side rails down, to attend to an emergency in another room. What legal principle is involved in this situation? Abandonment Reckless endangerment Negligence Battery

1) Abandonment involves refusing to care for a patient. 2) See 3). *3) Negligence involves failure to use prudent action or to follow the standard of care. 4) Battery is causing harm.

Management of Patient Centered Care An RN is assigned to complete a patient admission 30 minutes before the end of the RN's work shift. The RN verbalizes the inability to complete the assignment and to leave work at the scheduled time. Which action should be taken by the RN to resolve this problem? The RN works overtime to complete the task assigned. The RN delegates to co-workers the tasks needed to complete the assessment. The RN leaves the admission details for the next RN caring for the patient. The RN discusses options to best complete tasks with the nurse manager.

1) Accommodating the situation does not help solve the problem. 2) Patient admission is more than a series of tasks to be performed. The RN needs to discuss with the nurse manager the best method to accomplish the task. 3) Avoiding the situation by leaving the work for the next shift does not resolve the issue and can cause future issues within the team. *4) Conflict resolution should be attempted by the nurse manager and the RN. Collaboration is the preferred method of conflict resolution.

Ethical, Legal, Regulatory, Professional Implications An RN's expertise in a nursing specialty is recognized by which process? Accreditation Certification Licensure Registration

1) Accreditation applies to organizations, not to individual RNs. *2) Certification is provided by a nongovernmental authority, usually a professional organization, and may recognize expertise in a specialty. 3) Licensure is a governmental recognition of minimal competency in the profession, not a recognition of specialization. 4) Registration is similar to licensure in identifying those qualified to practice generally, not within a specialization.

Ethical, Legal, Regulatory, Professional Implications Which statements indicate the novice RN understands the significance of the NCLEX-RN examination for licensure as an RN? (Select all that apply.) The NCLEX-RN exam provides certification for Advanced Practice Nursing Employers are able to compare the knowledge level of applicants who have passed the NCLEX-RN examination. Passing the NCLEX-RN examination protects RNs from legal suits for malpractice by ensuring minimum competence. Passing the NCLEX-RN exam protects the public by establishing that minimum knowledge for competent practice has been demonstrated. The NCLEX-RN exam provides a standardized, objective test allowing for the possibility of relicensing in other states without having to take a different test.

1) Additional education and testing is required for Advanced Practice Nursing such as certification in Midwifery or as a Nurse Practitioner. 2) The NCLEX-RN exam is Pass-Fail so there is not a way for an employer to compare grades of applicants who have passed the test. 3) While passing NCLEX-RN indicates minimum competence, RNs are still at risk of malpractice lawsuits if they provide care which is unethical or negligent. *4) The NCLEX-RN exam is an objective, criterion-referenced test that protects the public by ensuring that candidates demonstrate a minimum level of knowledge for safe and competent care by entry level practitioners. *5) All states use the NCLEX-RN exam as a requirement for licensure and many states have mutual reciprocation agreements for ease of relicensing.

Professional Role Development An RN recognizes a discrepancy in a health care provider's medication order for a patient. What should the RN do to effectively resolve the discrepancy? Administer the medication as ordered and document concerns in the medical record. Change the order after consulting the pharmacist about the problem. Contact and communicate with the health care provider about the discrepancy. Complete a medication incident report and submit to the quality committee.

1) Administering a questionable medication is not following the competency of "safely administering medications." 2) Medication orders are not in the scope of nursing practice so this is not an appropriate action. *3) Safely administering medications is a core competency. 4) This would not resolve the discrepancy. Quality committee would focus on preventing similar problems in the future.

Professional Role Development What is a primary reason for engaging in continued professional education for most RNs? An RN's salary greatly increases with further education. RNs obtaining an advanced degree will have more power to influence change in the nursing profession. RNs must have a the minimum of a master's degree in order to be respected in the nursing profession. Education helps the RN to maintain clinical competency.

1) An RN's salary does not necessarily increase as further education is obtained. 2) Education does not always translate to increased power to influence change. 3) RNs do not need to have a master's preparation to be respected. *4) Lifelong learning helps the RN to stay abreast of current best practices for clinical competency.

Ethical, Legal, Regulatory, Professional Implications How should an RN address a patient's financial concerns when planning nursing care? Assess the patient's coping mechanisms. Seek to re-establish the patient's sense of control. Guide the patient to appropriate counseling or therapy groups. Refer the patient to appropriate institutional and community resources.

1) Assessing a family's coping mechanisms is not directly relevant to addressing financial concerns. 2) Seeking to re-establish a sense of control over financial matters is not within the RN's scope of practice. 3) Counseling or therapy groups may come at a cost and are unlikely to address a patient's financial concerns. *4) When a patient requires financial support during times of illness, an RN uses direct questioning and careful listening to direct the patient to appropriate institutional and community resources.

Management of Patient Centered Care Increased use of electronic health records poses a potential risk to which patient right? Autonomy Accountability Privacy Justice

1) Autonomy is the act of being independent and self-governing as a professional. 2) Accountability, while important, is more the responsibility of the RN to the patient and not a patient right. *3) The Patient Care Partnership (formerly known as "A Patient's Bill or Rights," recognizes the need for state and federal laws and hospital operational policies to protect the privacy of a patient's health information. This becomes increasingly challenging as medical records are placed on line where access to many people becomes much easier. 4) Justice is the obligation to be fair to all people. It does not really impact access to the electronic health record.

Management of Patient Centered Care Which situation would an RN know to report as a sentinel event? A back strain reported by a staff nurse after assisting a patient from bed to chair An error in which the wrong medication was administered but where no patient harm resulted A severe head injury sustained by a patient after becoming confused and falling down in a progressive care unit A patient requiring hospital readmission within two weeks of discharge for the same problem

1) Back strain is not a serious injury so it would not be a sentinel event. 2) An error which does not result in death or serious physical or psychological injury is not a sentinel event. *3) A severe head injury is "an unexpected occurrence involving death or serious physical or psychological injury or risk thereof," making it a sentinel event. 4) Readmission rates are a core measure of The Joint Commission (TJC) but are not sentinel events.

Management of Patient Centered Care A staff of RNs is expected to follow up on medication errors by reviewing incident reports and patient records to identify and correct problems with following the proper protocol for administering medications. What is this process called? Benchmarking Case management Root cause analysis Evidenced-based practice

1) Benchmarking is setting an acceptable level for the expected outcome of a process or action. 2) Case management is the use of outcome-based protocols for patient care designed to provide systematic care, which is high quality and cost effective. *3) Root cause analysis is the process of tracking events leading to errors to identify faulty systems and processes. 4) Evidence-based practices are protocols supported by research studies.

Professional Role Development "The profession of nursing is committed to institution-wide behavior patterns that embrace inclusivity and respect for individual differences." Which core competency does this statement represent? Caring Integrity Diversity Excellence

1) Caring is the fundamental consideration of the whole person. It does not specifically address individual differences. 2) Integrity is evident when organizational principles of open communication, ethical decision-making, and humility are encouraged, expected, and demonstrated consistently. It does not specifically address individual differences. *3) A culture of diversity embraces acceptance, respect, and inclusivity. While diversity can be about individual differences, it also encompasses institutional and system-wide behavior patterns. 4) Excellence is commitment to continuous growth and improvement. It does not specifically address individual differences.

Management of Patient Centered Care An RN approaches a nurse manager with a plan on how to manage unit resources more efficiently. Which of the following would exemplify an ineffective use of resources? Using a consistent method to communicate patient needs Canceling a diet tray for a patient who is to be discharged Having an extra supply of intravenous (IV) control devices on the unit Providing an unplanned teaching session for a patient and the patient's family

1) Concise and clear communication facilitates proactive planning of nursing care. 2) Canceling extra food trays saves money and resources. *3) Keeping extra supplies on the unit may increase the time another unit uses to find equipment needed for patient care. 4) An unplanned or unexpected teaching session is part of the job responsibilities of an RN.

Management of Patient Centered Care Which activity demonstrates fiscal responsibility by an RN? Overlooking the repeated tardiness of a coworker Keeping extra pillows stocked in a patient's room Promoting hand hygiene with patients and visitors Supplying a patient with incontinence pads for home use

1) Cost awareness is having an understanding of budgetary constraints related to salaries and resources. Ignoring repeated tardiness is a financial loss to the institution. 2) An RN has to be aware of cost containment. Placing extra supplies in patient rooms at the beginning of the shift can be wasteful. *3) RNs have to be able to provide high-quality, safe, and responsible care to patients. Prevention of hospital-acquired infection is being quality-and fiscally-responsible by decreasing costs associated with health care acquired infections. 4) An RN has to be aware of cost containment. Giving additional supplies to patients on discharge increases the costs of the unit and decrease available supplies for the patients on the unit.

Professional Role Development Which is an example of the RN engaging in critical thinking in the clinical setting? The RN does not administer medication on a patient who is nauseated, and notifies the health care provider. The RN places nonskid socks on the patient's feet prior to assisting the patient to the bathroom. The RN places a patient who had a stroke and is on swallowing precautions in an upright position before mealtime. The RN encourages a post-operative patient with an elevated temperature to take deep breaths and cough.

1) Critical thinking is about problem solving, not task orientated (reporting, executing physician orders). 2) Critical thinking is about problem solving, not task oriented (reporting, executing physician orders). 3) Critical thinking is about problem solving, not task orientated (reporting, executing physician orders). *4) Critical thinking incorporates multiple facets of knowledge while prioritizing care.

Professional Role Development Which statement made by an RN indicates a clear understanding of the meaning of critical thinking? "Critical thinking is making emotional decisions." "Critical thinking means considering all options." "Critical thinking is about how I feel about a patient." "Critical thinking is about judging others based on their religion."

1) Critical thinking relies on objectivity, not emotion. *2) Critical thinking rationally looks at all options, as objectively and systematically as possible. 3) Critical thinking suspends feelings for rationality. 4) Critical thinking does not use personal judgment.

Professional Role Development An RN is planning a wellness presentation focusing on primary prevention for children in public schools. Which nursing leader initiated the field of public health nursing? Lavinia Dock Lillian Wald Annie Goodrich Isabell Hampton Robb

1) Dock wrote the first nursing texts on the history of nursing. *2) Many child health and wellness programs in use today are based on Lillian Wald's efforts. Current proposals for health-care reform include her ideas about public health nursing, independent clinics, and health maintenance. 3) Goodrich was an RN educator who focused on nursing theory as well as hands-on practice. 4) Robb started the ANA and worked to organize the profession.

Ethical, Legal, Regulatory, Professional Implications What general guideline should not be followed by an RN, in documenting a patient's medical record to avoid legal action? Record nursing interventions and the patientâ s response. Record observations made by the RN. Identify inadequate staffing as a reason why patient treatment was not done. Record specific family member behaviors as they occur.

1) Documentation in the patient's medical record should be clear, concise and specific, and include each patient response to particular nursing actions. 2) Documentation should reflect only what the RN observed, not other health care providers' observations. *3) Issues about staffing or other system problems should not be addressed in the patient's record. Institutions do have policy/procedures to address these concerns. 4) Documentation in the medical record would describe objective behavior amongst family members, not make assumptions or subjectively interpret that behavior.

Professional Role Development During which stage of the nursing process does the RN analyze outcome information to determine effectiveness of care? Diagnosis Planning Evaluation Implementation

1) During the diagnosis phase of the nursing process an RN identifies patterns or clustering data, validates diagnosis, and formulates nursing diagnosis statements. 2) During the planning phase of the nursing process an RN establishes priorities and expected patient outcomes, plans nursing interventions, and writes an individualized plan of care. *3) During the evaluation phase of the nursing process an RN reviews patients goals and expected outcomes to determine whether they were measurable, realistic, and appropriate for resolving the patient's problems. 4) During the implementation phase of the nursing process an RN reassesses, sets priorities, performs or delegates nursing interventions, and documents actions.

Management of Patient Centered Care An RN knows that evidence-based practice in nursing requires a transition from nursing care based on opinions, past practices, and tradition to a practice based on which factor? Patient preferences and satisfaction Scientific research and proven substantiation Homeopathic medicine based on the nursing process Standardized practices preapproved by private health care insurance

1) EBP does not require a transition to nursing care that is based on patient preferences and satisfaction. *2) EBP does require a transition from nursing care that is based on opinions, past practices, and tradition to a practice based on proven evidence and scientific research. 3) EBP does not require a transition to nursing care that is based on homeopathic medicine. 4) EBP does not require a transition to nursing care that is based on standardized practices preapproved by private insurance and Medicare.

Management of Patient Centered Care A hospital asks RNs to evaluate the effects of new technology on improving patient care. Which is an advantage of using an electronic health record (EHR) system over using traditional systems? EHR systems are simpler to learn than most paper-based systems. EHR systems provide reminders about incompatibilities of medication or variances from normal standards. EHR systems require decisions about who can enter the data into the system and when entries should be made. EHR systems protect patient privacy more effectively than paper systems.

1) EHR systems are usually not simpler to learn than paper-based systems. *2) This is an advantage of EHR systems. 3) This is not an advantage of using EHR systems. 4) EHR systems do not typically protect patient privacy more effectively than paper systems.

Professional Role Development An RN is attending a conference about incorporating evidence-based practice into their specialty. Why is it important for the RN to participate in this professional activity? The RN is going to conferences because it is important for promotion to the charge nurse manager position. This RN is recognizing evidence-based practice as a new trend and they must learn about it. The RN will be able to interact with old friends from college and talk about their post-school experiences. The RN will have an opportunity to expand knowledge and provide input for future research on the unit.

1) Earning CEU and more experience will help with the promotion and application of the knowledge, but it is not the requirement for the charge nurse position. 2) Evidence based practice is not a new trend in the practice. Its principles are taught in nursing schools and incorporated in the curriculum. 3) It is good to see colleagues build the professional circles and exchange experiences but it is not the main purpose of the conference meetings. *4) Constant lifelong learning and participation activity demonstrates that the RN continually builds on a strong knowledge base for professional practice.

Professional Role Development What Quality and Safety Education for Nurses (QSEN) competencies are RN students expected to demonstrate, upon graduation from a pre-licensure program? (Select all that apply.) Efficiency Effectiveness Patient-centered care Quality improvement (QI) Evidence-based practice (EBP)

1) Efficiency: translating research into practice to increase access and to decrease costs is a Agency for Healthcare Research and Quality's (AHRQ) research focus area. 2) Effectiveness: improved health outcomes by using evidence to make informed health- care decisions costs is an Agency for Healthcare Research and Quality's (AHRQ) research focus area. *3) This is a Quality and Safety Education for Nurses (QSEN) competency. *4) See 3). *5) See 3).

An RN questions the nurse manager about why a traditional nursing practice is still being utilized on the unit, and references a nursing journal article supporting a more effective practice that could be employed. Which feature of professional nursing, as highlighted by the American Nurses Association's support of evidence based practice (EBP), is this RN demonstrating? 1) Applying widely accepted traditional nursing practice to patient care 2) Assuring safe, quality patient care using the best available evidence 3) Maintaining basic features of professional nursing knowledge 4) Distributing EBP findings to clinical nurses who lack research understanding

1) Entrenched nursing practices remain widely accepted and are barriers to application of EBP. *2) The interrelationship between clinical practice and EBP is evident when nurses question traditional practice and use the best available evidence to drive safe, quality patient care. 3) Nursing knowledge should be advanced through scholarly inquiry. 4) While providing information to those who lack understanding of EBP in clinical practice is helpful it does not ensure it is applied to patient care.

Ethical, Legal, Regulatory, Professional Implications Which ethical principle assists the RN in caring for a patient whose cultural practices conflict with the therapeutic health plan? Fidelity Veracity Autonomy Nonmaleficence

1) Fidelity is the obligation to be faithful to commitments made to self and to others. Commitments do not enter into this case. 2) Veracity requires the health-care provider to tell the truth and not intentionally deceive or mislead. This is not directly relevant to this case *3) Autonomy is the right of self-determination even if the health care provider does not agree. The RN must respect the patient's autonomy to choose to maintain cultural practices. 4) Nonmaleficence requires health care providers do no harm intentionally or unintentionally. In this case, if the patient chooses not to follow the therapeutic health plan it is not the nurse that is causing harm, so nonmaleficence does not apply in this case.

Professional Role Development Which reform did Florence Nightingale contribute to nursing practice? New surgical procedures Community health nursing Use of research to improve patient care Development of the interdisciplinary team

1) Florence Nightingale did not contribute new surgical procedures. 2) The growth of community nursing originated with the deaconesses of the Early Christian Church, before Florence Nightingale. *3) Florence Nightingale is referred as the first nurse researcher, drawing conclusions that today would be termed evidence-based nursing research. 4) The development of the interdisciplinary team in which each individual has duties and functions has ancient origins.

Professional Role Development Which activity does the National Council of State Boards of Nursing (NCSBN) use to assess current nursing practice? It reviews and revises the current pool of NCLEX test questions. It surveys newly licensed nurses about specific nursing care activities in daily practice. It visits major healthcare institutions nationwide to determine best practices in nursing care. It conducts focus groups of expert nurse educators with from different practice settings.

1) Following statistical analysis, the NCLEX test questions will be reviewed and revised for accuracy of content in current practice. However, this process assesses the exam, not current practice. *2) The most important part of responding to current practice is to learn from new nurses what is current clinical practice. This step is an essential activity of the NCSBN. 3) This activity establishes current practice in major health care centers and does not reflect practice among midrange and small health care institutions, rural and community practice. 4) This practice provides information about nursing education and may not provide enough information to establish current clinical practice from the viewpoints of new nurses.

Ethical, Legal, Regulatory, Professional Implications Which methods are likey to help the RN prevent patient injury, death, and lawsuits? (Select all that apply.) Maintaining a personal distance from patients Documenting or charting the patient care provided Keeping one's knowledge and nursing skills current Being indirect and avoiding problems with the patient Effective communication among interprofessional team members

1) Having a rapport with patients can help prevent lawsuits; maintaining personal distance may cause family members to see the RN as uncaring. *2) Documented health records are objective evidence that can flag potential problems before they occur. *3) Keeping knowledge and skills current can prevent injury, death, and lawsuits based on failure to follow current practice. 4) It is best to deal with problems promptly; errors can occur when problems are avoided. *5) Lack of effective communication is the leading cause of health-care-related errors leading to injury, death, and lawsuits.

Ethical, Legal, Regulatory, Professional Implications RNs are required to recognize and refer potential organ donors to local procurement organizations. Which factor initially influences the way an individual RN views this responsibility? Hospital policies and procedures Personal and professional values Educational preparation and experience Professional organizations' guidelines

1) Hospital policies and procedures may change the RN's existing views, but these views will have been initially formed on the basis of the RN's personal and professional values. *2) Professional values provide the foundation for nursing practice and guide the nurse's interactions with patients, colleagues, and the public. 3) Educational preparation and experience may change the RN's existing views, but these views will have been initially formed on the basis of the RN's personal and professional values. 4) External guidelines may change the RN's existing views, but these views will have been initially formed on the basis of the RN's personal and professional values.

Management of Patient Centered Care A group of RNs on the quality improvement (QI) committee identify injury from falls as a priority problem in a skilled nursing facility. What would be the next step to take in resolving this issue? Research evidence-based practices that prevent falls. Develop a competency-based staff in-service on the prevention of falls. Investigate the circumstances of the falls that have occurred in the facility. Implement standard practices recommended by the Institute for Medicine (IOM).

1) Identifying causes of the problem would occur before researching practices to address the situation. 2) Developing staff education would only be appropriate after it was determined that staff knowledge was determined as a contributing factor to the problem. Data must be collected and analyzed before determining education is the solution and what to include in the education. *3) After identifying a problem, the next step in resolving a problem is to identify the circumstances of the incidents where the problem occurred to determine the etiologies or causes (in this case investigating the falls) that have occurred. 4) Implementing a solution to a problem would be a later step in the process, after assessing contributing factors, analyzing causes, and setting goals.

Ethical, Legal, Regulatory, Professional Implications In which situation might the Good Samaritan Act provide a defense against potential legal action towards an RN? The RN arrives at an accident scene as a member of the emergency response team. The RN fails to report another health care providerâ s incompetency related to alcohol dependency. The RN performs a tracheotomy on a person choking and the person survives. The RN provides cardiopulmonary resuscitation and the patient experiences fractured ribs.

1) If the RN works as the member of the emergency response team, the RN's actions are guided by emergency protocols. 2) See 4). 3) See 4). *4) Good Samaritan laws are meant to encourage anyone in the medical profession to render assistance in an emergency situation, without fear of liability for negligence.

Professional Role Development A nurse educator is providing an in-service safety training to RNs to decrease falls and resulting injuries on nursing units. This is an example of which Institute of Medicine (IOM) competency? Interdisciplinary teamwork Evidence-based practice Quality improvement Patient-centered care

1) Interdisciplinary teamwork includes other types of care providers, not just RNs. 2) Evidence-based practice includes clinical expertise, patient values, and research in the decision making process. It might contribute to the content of training but would not be the best description of the training process itself. *3) Quality improvement measures health care quality and works to address issues. 4) Care providers partner with the patient and family to meet the patient's needs; the focus here is on safety from the point of view of what the RNs can do.

Management of Patient Centered Care An RN manager is informed that a staff RN is ill and will not be on the unit. Which initial action should the RN manager take to help ensure that patient care is not compromised? Take on a patient assignment and help other staff as needed. Assess the acuity level of the patients and the RN-patient ratio. Tell the nursing supervisor that a replacement RN will be necessary. Initiate functional nursing and use one RN to administer all medications.

1) It is not feasible that the RN manager take on a patient assignment prior to assessing the situation. In doing so, other duties normally assigned to the RN manager may be affected, and overall care may be compromised. *2) The RN manager needs to first assess the acuity level of the patients and the current RN-patient ratio to identify the next course of action. The stem asks for the initial action that the RN should take. 3) The RN manager must assess the situation before taking any additional action. The RN manager might then discuss the next course of action with the supervisor. 4) The RN manager cannot come to a conclusion to alter the type of nursing care delivery until an assessment is made of the current patient acuity and RN-patient ratio.

Management of Patient Centered Care What characteristics are evident in the RN who is known for the ability to effectively communicate and build strong teams? (Select all that apply.) Keeping silent on controversial issues Using active listening skills Using positive body language Providing constructive criticism in public Being sensitive to nonverbal communication cues

1) Knowing when to speak and when to be silent helps to build synergy and trust within a group, and fosters communication. *2) RNs with these abilities use active listening skills; know when to speak and when to be silent; use positive body language; are sensitive to nonverbal communication cues; and praise staff in public but give constructive criticism in private. *3) See 1). 4) Constructive criticism is best given in private. *5) See 1).

Management of Patient Centered Care Which situations represent the most common barriers to the implementation of research in practice? (Select all that apply.) An RN begins a research project but has a difficult time completing the project because of a lack of knowledge. The nurse manager and other nursing staff do not see the need for the research because the current practice is tradition. There are already findings from other projects that have not been implemented into practice. The RN proposes a research project to a nurse manager and the manager agrees to the project as long as the RN conducts the research unpaid and during non-work hours. The researchers do not understand issues from the bedside RN's point of view.

1) Lack of knowledge and understanding is not the most common challenge to implementing a research project. *2) This is an example of a barrier to research known as entrenched practices. 3) Experts often ask why there is a need to conduct more research when there is already so much research not being implemented into practice. This creates a barrier for support of research, but it is not the most common challenge to completing research in clinical practice. *4) This is an example of a barrier to research due to budget constraints, insufficient authority, and time available to conduct research. It is the most common challenge to completing research in clinical practice. *5) If the researcher does not understand practice issues, the reseach finding may not provide practical solutions for the bedside nurse.

Ethical, Legal, Regulatory, Professional Implications What is a primary difference between mandatory licensure and certification within the nursing profession? Licensure is regulated by professional organizations and the state board of nursing grants certification to practice in that state. Only the state board of nursing in a given state regulates licensure within that state; various professional organizations may grant certification to nurses. Individual certification status is only available for advanced practice nurses. On the other hand, mandatory licensure at the appropriate training level is required for all nurses in every state. Certification recognizes the attaining of knowledge or skills greater than the minimal level of competence that is required for licensure, but it does not provide the license itself.

1) Licensure is granted through the state board of nursing within the individual state and certification is granted through professional organizations. 2) Although the state board of nurses regulates licensure within the state and certification may be accomplished through various professional organizations, it is not the primary difference between licensure and certification. The primary difference is the level of competence of the difference is status. 3) Individual certification is granted through professional organizations for all professional nurses, who meet the requirements not just for advanced practice nurses. *4) Licensure is required to practice nursing and indicates that the individual has achieved minimal competence; certification indicates that the individual has achieved knowledge and skills that exceed minimal competence.

Professional Role Development Which was Lillian Wald's contribution to public health nursing? She founded nurse practitioner (NP) practice. She improved the standards of nursing education. She organized nurses to make home visits focusing on children's health. She improved hospital conditions, sanitation, nursing education, and health care.

1) Loretta Ford, not Lillian Wald, founded NP practice. 2) Isabel Adams Hampton Rob, not Lillian Wald, focused on standards of nursing education. *3) Lillian Wald began the practice of visiting nurses. 4) Florence Nightingale, not Lillian Wald, made these multiple improvements.

Professional Role Development Which behavior by an RN is in violation of nursing's professional boundaries? Providing a lower back rub to a patient in labor Taking a picture of a pediatric patient and posting it to social media Recommending a primary health care provider to a neighbor Assisting an unlicensed assistive personnel (UAP) in providing perineal care to a patient with a urinary catheter

1) Low back massage helps to ease back pain in a patient in labor. *2) An RN's use of social media can be a boundary violation if pictures or comments are posted without patient or guardian permission. 3) Recommending a primary health care provider to a neighbor is within the professional boundaries of an RN. 4) Foley and perineal care is part of nursing care as long as it is performed without sexual connotations.

Ethical, Legal, Regulatory, Professional Implications What is the best response an RN can give a patient who requests information on whether to go to the closest hospital or to travel to a magnet hospital for a needed surgery? "An area of strength in magnet hospitals is the ability to retain staff because of good benefits and working conditions." "A magnet hospital undergoes an external review documenting standards of nursing excellence, which results in higher, overall quality patient care." "A non-magnet hospital may simply have chosen not to seek the magnet designation; it may be as good, or even better than, a magnet hospital." "A magnet hospital allows nurses greater autonomy and focuses on evidence-based practice."

1) Magnet hospitals are able to retain nursing staff in part because of good working conditions but not necessarily because of good benefits. In any case, these facts only indirectly address the question of whether going the extra distance to a magnet hospital is worthwhile. *2) This is a true statement and it directly and meaningfully answers the question posed by providing information about higher quality patient care. 3) This is a true statement; however, it does not directly address the question of whether going the extra distance to a magnet hospital is worthwhile. 4) This is a true statement; however, it does not directly address the question of whether going the extra distance to a magnet hospital is worthwhile.

Professional Role Development A nursing mentor evaluates a new RN and notes that the new RN has appropriate skills, knowledge, attitudes, values, and abilities for practice. Which attribute is the mentor evaluating? Morality Competency Caring Professionalism

1) Morality refers to the concept of right and wrong, not knowledge and skills. *2) Competency refers to the combination of skills, knowledge, attitudes, values and abilities that support the safe and effective practice of an RN. 3) Caring is an aspect of emotional intelligence and is not necessarily related to skills and knowledge. 4) Professionalism refers to the behaviors and attitudes exhibited by an individual that are recognized by others as the traits of a professional. It is an outward attribute, rather than the underlying knowledge, skills, and abilities being evaluated in this example.

Ethical, Legal, Regulatory, Professional Implications Which elements must be present in order for a patient to make a claim of professional negligence against an RN? (Select all that apply.) A civil law was violated by the RN. Actual damages resulted from the RN's action. The RN did not perform to the standard of care. The RN was responsible for caring for the patient at the time of the injury. The RN's failure to act was the cause of the patient injuries.

1) Negligence can occur even if civil law has not been violated. *2) There were actual damages resulting from professional actions. *3) A failure to act which caused damages is considered negligence. *4) Within a professional relationship there is a duty owed to the patient by the RN assigned to the care. *5) Damage or injury must result from the RN's action for negligence to be claimed.

Professional Role Development What must the RN do to become certified by the American Nurses Credentialing Center (ANCC) in gerontological nursing? Obtain a master's degree. Passing an approved exam. Submit an application to the state board of nursing. Document clinical hours worked in a nursing home.

1) No advanced degree is required for this certification. *2) The individual seeking licensure must have demonstrated an essential amount of competency necessary to perform a unique scope of practice prior to receiving the licensure. 3) The application is made directly to the ANCC. 4) The clinical hours can be worked in any facility.

Ethical, Legal, Regulatory, Professional Implications How does the ethical principle of nonmaleficence apply to the practice of organ transplantation? People who will benefit society should be given priority to receive transplants. Providers of health care should do no harm in the process of organ transplantation. There should be an even distribution of organ transplants among those who need one. People who have the financial resources for an organ transplant should be able to have it.

1) Nonmaleficence is the prevention of intentional harm. It does not apply to prioritization of transplantation. *2) Nonmaleficence is the prevention of intentional harm. 3) Nonmaleficence is the prevention of intentional harm. It does not apply to the distribution of transplantation. 4) Nonmaleficence is the prevention of intentional harm. It does not apply to rights to transplantation.

Ethical, Legal, Regulatory, Professional Implications Which is the main function of a nurse navigator? To act as a mediator for the patient in dealing with health care insurance and provider billing. To assist hospitalized patients to develop a realistic plan for obtaining ongoing care following discharge. To help patients and families with specific types of complex care problems to get through the network of treatments. To work independently of the health care team to assist patients and families with getting the treatment required for a chronic health problem.

1) Nurse navigators do not typically engage in mediation. 2) The main function of a nurse navigator is to eliminate barriers and coordinate care. Nurse navigators would only help develop care plans within their scope of practice. *3) The nurse navigator, who usually is focused on one specialty area, works with patients and families with complex health care issues serving as a patient/family advocate in eliminating barriers and providing guidance through the maze of treatment. After referral from a health care provider, the nurse navigator contacts the patient and develops an individualized plan of care designed to help the patient progress more easily through treatment. 4) Nurse navigators work in close coordination with the health care team.

Ethical, Legal, Regulatory, Professional Implications What is one purpose of nurse practice acts? To establish nursing standards that govern the ethical conduct of nurses To hold nurses accountable to national standards of quality patient care To provide a framework that the legal system can use to determine whether a nurse has breached standards of care To define nursing standards based on specific health care settings policy and procedure manuals

1) Nurse practice acts govern standard of care, not ethical conduct. 2) Nurse practice acts operate at the state level, not the national level. *3) A state's nurse practice acts provides a framework for the court on which to base decisions when determining whether a nurse has breached a standard of care. Nurse practice acts identify grounds for disciplinary actions such as suspension and revocation of nursing licensure in each state. 4) Nurse practice acts are statewide, not specific to particular health care settings.

Ethical, Legal, Regulatory, Professional Implications An RN smells alcohol on a coworker's breath. Which action should the RN take? Discuss the incident with other colleagues. Confront the coworker about behavior. Report the incident to the nurse manager. Overlook the coworker's issue one time.

1) Opinions of other staff is not relevant. Failing to report another health-care providers problem can produce legal action. 2) The RN is required to report another's incompetence to the direct supervisor. *3) The RN is required to report another's incompetence to the direct supervisor, The RN is legally and ethically obligated to report this to protect not only the patient but the RN. 4) This would not protect the patient or the co-worker. The RN has an obligation to protect patients or can be sued for malpractice.

Professional Role Development What are the responsibilities of a novice RN who is participating in a wound prevalence study? (Select all that apply.) Publish results. Collect data. Advocate for patients. Practice within ethical guidelines. Maintain standards of nursing practice.

1) Participation in a wound prevalence study does not require the publication of results. *2) Nursing responsibilities are presented in seven major areas: 1. Modeling and valuing nursing 2. Professional growth 3. Standards of nursing practice 4. Nursing research 5. Professional stewardship and the advancement of nursing 6. Patient advocacy 7. Legal and ethical practice. *3) See 2). *4) See 2). *5) See 2).

Ethical, Legal, Regulatory, Professional Implications In which situation would the Good Samaritan Act protect the RN from possible litigation? (Select all that apply.) An RN performs a tracheotomy on a person with a partially obstructed airway. An RN sees a person fall from a ladder and applies an immobilizer to a fractured arm. An RN performing CPR breaks several of the personâ s ribs. An RN stops at a car accident and pulls the individual from the car without stabilizing the spine. An RN performs the Heimlich maneuver on an individual who is choking, but is unsuccessful.

1) Performing a tracheotomy is not in an RN's scope of practice. *2) Immobilizing a fracture is within the scope of practice of an RN. *3) CPR certification is within the RN's scope of practice. The rib fractures are a consequence of the CPR. 4) Neglecting to stabilize the spine in a trauma patient may be determined to be negligence. *5) Performing the Heimlich maneuver is within the RN's scope of practice. Even if the outcome is not positive, the RN is covered by the Good Samaritan Act.

Ethical, Legal, Regulatory, Professional Implications An RN unintentionally fails to apply for relicensure while still practicing. Which statement is accurate, concerning the legal implications of this error? Practicing without a license is a violation of civil law. Failure to renew a nursing license is an act of malpractice. Continuing to practice without a current license is a criminal act in all states. Continuing to practice without an up-to-date license is an example of an unintentional tort.

1) Practicing without a license is a violation of civil law. 2) Failure to renew a nursing license is an act of malpractice. *3) Failure to renew a nursing license is the most common nursing violation of criminal law. 4) An unintentional tort occurs when there is damage to a person or their property. Failure to renew a license does not directly result in damages to another party.

Professional Role Development Which event in nursing had the most significant impact on the development of the associate degree in nursing? A decline in associate degree programs in the United States during the 1980s An American Nurses Association (ANA) position paper on RN qualifications The establishment of the Organization for Associate Degree Nursing (OADN) A pilot associate degree nursing program that was initiated at Teacher's College in New York City

1) Programs for associate degree nursing were increasing in number in the 1980s, not decreasing. 2) The ANA position paper for entry into practice recommended the baccalaureate degree as the entry level into practice. *3) The Organization for Associate Degree Nursing was founded as the voice of associate degree nurses in 1984. 4) The pilot program was initiated in 1952 but did not have a significant impact on associate degree education.

What is the purpose of the American Nurses Association (ANA) Standards of Professional Nursing Practice? 1) To provide guidelines for nursing education 2) To establish requirements for nursing licensure 3) To improve the practice of nursing 4) To promote unity within the nursing profession

1) Providing guidelines for nursing education is the primary purpose of the National League for Nursing (NLN). 2) Nursing licensure requirements are enacted by state law. *3) The ANA Standards provide a means to evaluate the quality of nursing practice and assists in assuring the public that quality nursing care is being delivered. 4) See 3).

Management of Patient Centered Care Which are American Association of Colleges of Nursing (AACN) standards for establishing a healthy work environment? (Select all that apply.) Creating competition Skilled communication Meaningful recognition Adequate accommodation Effective decision making

1) Resolving conflict in a competitive manner usually means that a person wants to force the issue or achieve personal goals. This is a win-lose situation. *2) Skilled communication is one of the AACN's six standards for establishing and nurturing a healthy work environment. *3) Meaningful recognition is one of the AACN's six standards for establishing and nurturing a healthy work environment. 4) Accommodation smooths things out amongst co-workers, but often results in one party feeling like they have capitulated. The disadvantage of this strategy is that if it is used frequently, resentment and hostility can fester. Anger may also be on to displaced to people who have no stake in the issue, such as a patient, and this disrupts good care. *5) Effective decision making is one of the AACN's six standards for establishing and nurturing a healthy work environment.

Ethical, Legal, Regulatory, Professional Implications Which best expresses why RNs should initially reflect on and identify their personal values? To develop awareness of the values held by different generations To be value-neutral and to interact effectively with patients who have different values To gain insight into similarities between patients' religious beliefs and the nurse's beliefs To become more assertive and to develop independent functioning

1) See 2). *2) Nurses are encouraged to behave in "value neutral" and "nonjudgmental" ways in their professional lives, and separate their personal values from their professional duties in caring for patients, and not make judgments on a personal basis. 3) See 2). 4) See 2).

Management of Patient Centered Care What is the best response by an RN mentor when a novice RN asks the purpose of using nurse theory to direct nursing care? "Nursing theory is used for specific directions on how to provide care." "Nursing theory can be used to provide guiding principles." "Nursing theory defines the structure of the nursing department." "Nursing theory can provide a single purpose for practice."

1) See 2). *2) Nursing theories provide the underlying conceptual support that promotes consistent care. 3) See 2). 4) See 2).

Ethical, Legal, Regulatory, Professional Implications What is the difference in language used in describing the scope of practice for LPN/LVNs and RNs, as outlined in state nurse practice acts? (Select all that apply.) There is more dependent language used in the scope of practice for RNs. There is more permissive language used in the scope of practice for RNs. There is more restrictive language used in the scope of practice for LPN/LVNs. There is more independent language used in the scope of practice for LPN/LVNs. There is more identical language used in the scope of practice for both the RN and LPN/LVNs.

1) See 2). *2) Regulations, especially those with the purpose of defining the scope of practice of various health care professionals, can be written with either restrictive or permissive language. Restrictive language restricts the practitioner to performing only the functions and procedures outlined in the regulations. LPN/LVN practice acts are often written with restrictive language. Permissive language allows practitioners to use judgment and make decisions to serve the purpose of performing in their roles. Permissive language is used for the more autonomous practice of the RN. *3) See 2). 4) See 2). 5) The LPN acts use more restrictive language than RN acts, which has more permissive language. The LPN is in a dependent role; the RN is in a more independent role.

Ethical, Legal, Regulatory, Professional Implications Which are examples of tasks a nurse navigator would provide to a referred patient? (Select all that apply.) Delay required paperwork associated with care. Coordinate transportation for health care visits. Accompany a nervous patient to a clinic appointment. Withhold written summaries of health care treatments. Respond to questions about side effects of medications.

1) See 2). *2) The nurse navigator, who usually is focused on one specialty area, works with patients and families on complex health care issues, and serves as a patient/family advocate to eliminate barriers to and provide guidance on navigating the maze of treatment. After referral from a health care provider, the nurse navigator contacts the patient and develops an individualized plan of care designed to help the patient progress more easily through treatment. The nurse navigator is available for help 24 hours per day and is concerned with providing the patient a sense of empowerment, and helping to alleviate patient anxiety and depression during treatment. The nurse navigator may arrange transportation, assist with insurance and other forms, provide written summaries of care, give instructions for self-care, and answer questions about side effects of medications. *3) See 2). 4) See 2). *5) See 2).

Professional Role Development What is the primary focus of the National League for Nursing (NLN)? To accredit nurses in practice To improve the standards of nursing education To promote quality assurance for hospital nursing services To encourage collaboration among health care providers

1) See 2). *2) The primary focus of the NLN is to be a leader in nursing education, commitment to its members, to be a champion for nurse educators, and to advance the science of nursing education. 3) See 2). 4) See 2).

Ethical, Legal, Regulatory, Professional Implications What is the initial step an RN should take when confronted with an ethical dilemma? Gather additional data. Determine desired outcomes. Clarify the issue in question. Define the nurse's obligation.

1) See 3). 2) See 3). *3) The steps to resolving an ethical dilemma are: 1) identify and clarify the ethical problem, 2) gather factual data, 3) identify and evaluate options, 4) make a decision, and 5) act and assess. 4) See 3).

Management of Patient Centered Care A novice RN asks why an occurrence report is required after a potential patient error has been averted or missed. How should the RN mentor respond? "An occurrence report will be used to track employee injuries." "The report will direct disciplinary action of the staff involved." "The occurrence report will provide legal evidence in a lawsuit." "The report functions as a useful tool to identify areas of high risk."

1) See 4). 2) See 4). 3) See 4). *4) Incident reports improve the management and treatment of patients by identifying high-risk patterns and initiating in-service programs to prevent problems.

Ethical, Legal, Regulatory, Professional Implications Which statement best describes the purpose of the American Nurses Association (ANA) Standards of Practice? The standards provide outcome criteria for nursing specialties. The standards define protocols for nursing executives. The standards describe the protocols required by The Joint Commission (TJC). The standards provide criteria for evaluating the quality of nursing care.

1) See 4). 2) See 4). 3) The ANA standards do not describe The Joint Commission's protocols. *4) The ANA standards provide the criteria for evaluating the quality of nursing care.

Professional Role Development Which self-care activity prevents burnout? Attending a wine tour with one'sspouse. Catching up on gossip with friends. Taking an overnight trip to a nearby casino. Participating in regular aerobic exercise.

1) See 4). Strategies for problem-solving in stress management include avoiding drinking alcohol. 2) See 4). Strategies for problem-solving in stress management include avoiding toxic behaviors such as gossiping. 3) See 4). Strategies for problem-solving in stress management include avoiding toxic behaviors such as gambling. *4) A widely accepted definition of burnout is a state of emotional exhaustion that results from the accumulated stress of an individual's life. Although there are many schools of thought about preventing burnout, several common threads run through many of these theories. These views include setting personal goals, identifying problems, and using strategies for problem-solving. Exercising regularly is a strategy for problem-solving in stress management.

Ethical, Legal, Regulatory, Professional Implications The RN conducts an initial assessment on a hospitalized patient and only documents those elements that are abnormal or unusual. Which type of communication is this? Situation, Background, Assessment, Recommendation Charting by exception Nursing Minimum Data Set Evidence-based practice

1) Situation, Background, Assessment, Recommendation is a communication process that includes providing all information. *2) Charting by exception includes only abnormal finding and is a valid charting technique. 3) The Nursing Minimum Data Set includes more information than just unusual or abnormal elements. 4) Evidence-based practice is based on thorough documentation of all elements of effective care.

Professional Role Development An RN implements a fall risk protocol for a patient who has been experiencing dizziness and weakness. Which competency does this RN's action demonstrate? Spirit of inquiry Human flourishing Clinical judgment Professional identity

1) Spirit of inquiry is curiosity; it is not the primary competency called on when implementing a protocol. 2) Human flourishing involves helping people achieve fulfillment and self-actualization. Implementing a fall risk protocol does not directly demonstrate this competency. *3) This is an example of good clinical judgment; the determination of essential nursing care for safe patient outcomes. 4) Professional identity is associated with the RN's place in the profession; implementing a protocol is not a direct reflection of professional identity.

Professional Role Development What is the main purpose for periodic review of the American Nurses Association (ANA) Code of Ethics by the nursing profession? To ensure acceptability by other health care professions as the most comprehensive code of ethics To provide a decision-making process so that RNs in any health care setting can develop, learn, and practice sound ethical decision-making skills To reflect changes in standards of practice resulting from value modifications and technological advances in the nursing profession and society as a whole To update ethical situations, and disseminate additional information for RNs to help guide their decision making

1) The ANA Code of Ethics has been acknowledged by other health care professions as the most complete code of ethics. It is sometimes used as a benchmark against which other codes of ethics are measured. 2) Although RNs deal with problems related to the physical and psychological needs of patients, many feel inadequate when dealing with ethical problems associated with patient care. RNs in any health care setting can, however, develop the decision-making skills necessary to make sound ethical decisions if they learn and practice using an ethical decision-making process. *3) Ideally, codes of ethics should be reviewed periodically to reflect changes in values and technological advances in the profession and society as a whole. 4) An electronic format that contains embedded text to references, ethical situations, and other additional information for nurses to help guide their ethical decision-making are changes that the ANA made to the Code of Ethics for 2015 revised version.

Ethical, Legal, Regulatory, Professional Implications Which agency should an RN recommend a patient to contact, as a source for information about Affordable Care Act (ACA) regulations? Community Care Transitions Program (CCTP) State Consumer Assistance Program Local Accountable Care Organization (ACO) Center for Medicare and Medicaid Innovation

1) The Community-based Care Transitions Program (CCTP), created by Section 3026 of the Affordable Care Act, tests models for improving care transitions from the hospital to other settings and reducing readmissions for high-risk Medicare beneficiaries. It is not designed as a source of information about ACA regulations. *2) The Affordable Care Act (ACA) established a Consumer Assistance Program in each state to help citizens navigate the regulations of the ACA. These programs are not yet all funded and/or completely active but ideally, individuals with questions about ACA regulations would be referred to this agency for information. 3) Accountable Care Organizations (ACOs) are groups of doctors, hospitals, and other health care providers, who come together voluntarily to give coordinated high quality care to their Medicare patients. An accountable care organization is not designed as a source of information about ACA regulations. 4) The CMS Innovation Center develops new payment and service delivery models in accordance with the requirements of section 1115A of the Social Security Act. Additionally, Congress has defined (both through the Affordable Care Act and previous legislation) a number of specific demonstrations to be conducted by CMS. It is not designed as a source of information about ACA regulations.

Management of Patient Centered Care An announcement is made about a new hospital policy on lateral violence, in response to The Joint Commission (TJC) requirement. A staff member says they don't understand why the TJC is concerned with how staff treat one another. Which primary fact should the RN cite, in responding to the staff member? A good team increases productivity. Public dissent promotes lack of respect for health care workers. Policies regulating staff interactions facilitate the good of the system. Disruptive behaviors increase the risk of error.

1) The Joint Commission (TJC) has developed guidelines relative to lateral violence because there are no federal standards regulating workplace violence and existing state laws are unclear and/or difficult to enforce. Reasons for TJC's action are based on the fact that freedom from bullying, intimidation and other disruptive behaviors is essential for a culture of safety. Disruptive behaviors interfere with the function of the health care team and foster medical errors. They also contribute to poor patient satisfaction and preventable adverse outcomes. They increase cost and loss of qualified staff. It is true that good team work may increase productivity but the crucial concern is patient safety, which is intimately related to prevention of error. 2) Concerns about safety and quality are the focus of TJC's agenda when developing guidelines related to lateral violence. The public image of health care workers was not the identified concern. 3) Concerns about safety and quality are the focus of TJC's agenda when developing guidelines related to lateral violence. The public image of health care workers was not the identified concern. *4) TJC has developed guidelines relative to lateral violence because there are no federal standards regulating workplace violence and existing state laws are unclear and/or difficult to enforce. Reasons for TJC's action are based on the fact that freedom from bullying, intimidation, and other disruptive behaviors is essential for a culture of patient safety. Disruptive behaviors interfere with the function of the health care team and foster medical errors. They also contribute to poor patient satisfaction and preventable adverse outcomes. They increase cost and loss of qualified staff.

Which organization is responsible for ensuring that schools maintain quality across nursing education programs? 1) The Joint Commission 2) American Nurses Association 3) National League for Nursing Accrediting Commission 4) National Council of State Boards of Nursing

1) The Joint Commission accredits hospitals based on a number of specific patient safety criteria. 2) The American Nurses Association is concerned with nursing practice and certification. *3) The National League for Nursing Accreditation. 4) Nursing licensure is a state right and is not dependent upon elected political power.

Management of Patient Centered Care Which scenario is an example of an RN implementing the Roy Adaptation model into clinical practice? A mental health unit RN offers a stress management class to assist all patients in implementing stress management techniques. An RN analyzes patient data using nursing diagnoses to determine the patient's ability to deal with a urinary tract infection. A community health RN refers a patient who is experiencing withdrawal symptoms from narcotics to a drug rehabilitation facility. An RN asks a family of a patient who is deceased if they have any spiritual or cultural requests related to the patient's funeral arrangements.

1) The King Model of Goal Attainment believes the RN can provide care in groups to attain a common goal. *2) Incorporating the use of nursing diagnoses into care is an example of using the Roy Adaptation Model. 3) Referring a patient to rehabilitation would be an example of Orem's Self Care Model where nursing is based on the belief that health care is each individual's own responsibility. 4) The Watson Model of Human Caring takes into consideration the patient's family and their spiritual beliefs.

Professional Role Development Which Quality and Safety Education for Nurses (QSEN) competency is an RN demonstrating when reviewing patient falls in an effort to decrease patient injury? Interdisciplinary teamwork Quality improvement Patient-centered care Informatics

1) The QSEN competency on interdisciplinary teamwork deals with basic organizational and systems leadership for high-quality care and patient safety. *2) The QSEN competency on quality improvement (QI) deals with information management and application of patient-care technology. 3) The QSEN competency on patient-centered care deals with scholarship on patient-centered care. 4) The QSEN competency on informatics deals with health-care policy, finance, and regulatory environments, interprofessional communication and collaboration for improving health outcomes, clinical prevention and population health, and professionalism and professional values.

Management of Patient Centered Care Which organization developed a framework to optimize team performance across health care delivery systems? Quality Improvement Organization (QIO) Institute for Healthcare Improvement (IHI) Quality and Safety Education for Nurses (QSEN) Agency for Healthcare Research and Quality (AHRQ)

1) The Quality Improvement Organization (QIO) is a federal program designed to review medical care, verify its necessity, and assist Medicare and Medicaid beneficiaries with complaints about quality of care. 2) The Robert Wood Johnson Foundation and the Institute for Healthcare Improvement (IHI) joined together to create a framework called Transforming Care at the bedside to institute change on medical surgical units. The goals was to improve care and staff satisfaction. 3) The Quality and Safety Education for Nurses (QSEN) is a project developed by the Robert Wood Johnson Foundation to improve the quality and safety of patient care by focusing nursing education on student competency. *4) The Agency for Healthcare Research and Quality (AHRQ) TeamSTEPPS is an evidence-based framework to optimize team performance across health care delivery systems.

Professional Role Development A new patient arrives on the unit and an RN and LPN begin the patient's assessment. Which task can the LPN peform? Establishing priorities for patient care Establishing diagnoses Setting long-term goals Contributing to the development of a care plan

1) The RN determines priorities. 2) The RN establishes diagnoses. 3) The RN develops an implementation plan with short- and long-term goals. *4) The LPN can contribute to development of care plans.

Management of Patient Centered Care An RN delegates the task of placing the telemetry monitor to the unlicensed assistive personnel (UAP). The UAP states, "I've never done this before." What should the RN say to the UAP? "Ask another UAP to show you." "I'll ask someone else to do it." "I'll go with you and show you how to do this." "It's easy. Just follow the directions on the package."

1) The RN is responsible for communicating clear instructions when delegating. 2) Although this accomplishes the task, it does not help the person who needs the information. *3) The RN recognizes the need to educate other health care workers and offers the needed hands-on instruction. 4) The RN role includes instruction and assistance.

An RN notifies the health care provider after making an error in medication administration. There were no adverse patient outcomes as a result of the error. Can the RN be sued for malpractice by the patient? 1) Yes. The RN did not follow hospital protocols. 2) Yes. The RN failed to take follow up action. 3) No. The patient was not harmed. 4) No. The RN notified the health care provider.

1) The RN may not have followed hospital protocols however no harm was done to the patient. 2) The RN did follow-up after the error was made. *3) The four elements of duty; breach of duty; caustation and damage or injury must be present to be held legally liable. 4) Notifying the physician/provider does not exempt the RN from liability.

Ethical, Legal, Regulatory, Professional Implications An RN's values conflict with a patient's beliefs about health care. How does the RN resolve this conflict? Explain the reasons why care needs to be provided according to protocol. Include the patient's spiritual preferences into the plan of care. Guide the patient to appropriate counseling or therapy groups. Refer the issue to the health care provider for resolution.

1) The RN needs to explore the personal value system and the patient's beliefs. Explaining protocols does not address the conflict. *2) When an RN needs to address a conflict between a patient's ability to participate in their care and the RN's ability to give care because of the patient's spirituality the RN demonstrates openness to other's ideas by knowing and being comfortable with their spirituality; incorporates the patient's spiritual preferences in the plan of care to assist with religious beliefs; and/or incorporate spiritual practices and symbols into the plan of care to promote the health of the patient by respecting their belief system. 3) Early psychosocial interventions in the form of counseling or therapy groups may be helpful to the patient, but will not resolve the conflict. 4) Referring the issue to another does not help to resolve the conflict.

Management of Patient Centered Care An RN is planning an in-service on how the unit can reduce the risk for falls for patients. Which organization's resources would be most helpful to the RN? The Joint Commission (TJC) Institute of Medicine (IOM) American Nurses Association (ANA) Agency for Healthcare Research and Quality (AHRQ)

1) The TJC is an agency that sets mandatory National Patient Safety Goals addressing particular risks for patients. Hospitals improve quality and safety by making these goals a priority in patient care. TJC would not be helpful to the RN's efforts for providing an in-service on Risk for Falls. 2) The IOM is an agency that published a report about the future of nursing and the need for RNs to be educated on, collaborative in, and responsive to the rapidly changing health care system. They do not provide information about immunizations. 3) The ANA is an organization that represents the interests of RNs in a variety of work settings. They also publish a Code of Ethics. They do not provide information about immunizations. *4) The AHRQ uses quality indicators as measures of health care quality from easily accessible inpatient hospital administrative data. Quality indicators (QI) include prevention, inpatient, patient safety, and pediatric information. The QIs are used to focus efforts on potential quality concerns so they may be addressed by further investigation.

Professional Role Development During an employment interview, a novice RN is asked to provide a personal example of how autonomy can be displayed in nursing practice. Which response would demonstrate that the novice RN understands autonomy in the nursing profession? "With the proper training and resources, I can be autonomous." "I can work independently and willingly take responsibility for my actions." "I work well with others and have been in management positions in the past." "Being autonomous is a challenge because of the hierarchal nature of nursing."

1) The answer places no conditions on accepting responsibility for one's own actions. *2) Autonomy means that the individual has self-determination over functions within the workplace. A professional must be responsible and accountable for her or his actions. By definition, "autonomy" means that the individual has self-determination over functions within the workplace. A professional must be responsible and accountable for her or his actions. RNs have had more difficulty achieving autonomy because of the hierarchal nature of nursing organizations and the continued dependence on the medical profession. 3) This does not answer the question of being responsible for one's actions. 4) This is true, but the question is about the individual's ability to be autonomous.

Management of Patient Centered Care A unit has high turnover and low staff morale because of patient acuity. The nurse manager asks an RN to act as a change agent for the department. Which attribute does the RN need to be a change agent? Understanding of the causes of patient acuity. Experience in talking to different kinds of people Ability to implement conflict resolution Consistency in thoughts, beliefs, and attitudes

1) The causes of patient acuity may not be able to be changed, and understanding them is not a requirement to be the change agent. 2) Having the ability to talk to people is not enough to encourage change. *3) Integrative thinking means the ability to retain a big picture focus while dealing with parts of the problem. 4) Being consistent in thoughts, beliefs, and attitudes is good, but one also must have flexibility to adjust to changes.

Ethical, Legal, Regulatory, Professional Implications Which is a factor causing the US to be ranked lower than other industrialized countries on key measures of health care outcomes? The US has greater population diversity than other industrialized countries. The range of lifestyle choices is broader in the US than in other industrialized countries. Access to health care is less widespread in the US than in other industrialized countries. The percentage of older people is rising faster in the US than it is in other industrialized countries.

1) The major factor related to health care and health outcomes which differentiates the US from other industrialized countries is access to care, not population diversity. 2) The major factor related to health care and health outcomes which differentiates the US from other industrialized countries is access to care, not range of lifestyle choices. *3) The major factor related to health care and health outcomes which differentiates the US from other industrialized countries is access to care. 4) The major factor related to health care and health outcomes which differentiates the US from other industrialized countries is access to care, not the rate of increase of elderly in the population.

Professional Role Development What is the primary focus of the National Council of State Boards of Nursing (NCSBN)? To promote excellence in nursing education To promote professional growth and development for all nurses To provide regulation of nursing practice to ensure public safety To conduct research to identify evidence-based best clinical practices

1) The primary purpose of the National League for Nursing (NLN) is to maintain and improve the standards of nursing education. 2) Professional growth and development of nurses is a primary purpose of the American Nurses Association. *3) The mission of the NCSBN is to provide leadership in regulation of nursing practice to protect the public. 4) Many nursing organizations and healthcare institutions conduct research to determine best practices. While the NCSBN supports evidence-based clinical research, it is not their primary focus.

Management of Patient Centered Care A patient newly diagnosed with a disease asks an RN how to determine whether a Web site article has reliable information about self-management of the disease. Which rule of thumb should the RN's response include? The most useful research comes from expert researchers sponsored by the pharmaceutical industry Authors who list their credentials as a physician, nurse practitioner, or RN are the most trustworthy. Articles with crowd-sourced reviews and contributions, such as those on Wikipedia, are most likely to be current. Information from a Web site owned by a government or educational agency is most likely to be of high quality.

1) The sponsor of a Web site or article should be reviewed with caution, as there may be prejudice or bias in the recommendations. 2) The author's credentials should be listed; however, identifying one's self as a health care provider or RN does not guarantee the data is scholarly. 3) With crowd-sourced reviews, quality may vary, as there is no professional oversight of what is posted. *4) Government and educational Web sites are less likely to be biased and more likely to include recognized professional and peer reviewed sources that provide a higher degree of accuracy and reliability.

A graduate RN asks an unlicensed assistive personnel (UAP) to report changes in a patient's urine output. What aspect of this delegation indicates a violation to the rights of delegation, and requires that the graduate RN follow up on the request? 1) The UAP does not have the proper qualification to complete this particular task. 2) The instruction should have included a request for specific information on when and what the UAP should report. 3) This task can be completed independently by the UAP and does not need the graduate RN's judgement. 4) The UAP has 5 years of experience but was assigned to the current unit two months prior.

1) The task is a relatively simple one and within the skill set for delegation. *2) The right communication was violated. The RN should have included information to report such as low or high volumes and a time frame to provide the information. 3) This is a relatively simple, low-risk procedure that can be delegated. 4) This is a common task and assignment to a new unit over the last few months would not constitute a violation of the rights of delegation.

Ethical, Legal, Regulatory, Professional Implications What legal violation does the RN commit when giving a medication to the patient who refuses it? Assault Intentional tort Malpractice Professional misconduct

1) The unjustifiable attempt to touch another person or the threat of doing so constitutes assault. *2) A planned intended outcome of the RN's action may result in physical or emotional harm to the patient, and shows intentionality. 3) Liability resulting from improper practice is not reflected in this act, although the moral dimensions of the act are questionable. 4) Professional misconduct indicates unreasonable lack of skill in performing professional duties.

Ethical, Legal, Regulatory, Professional Implications How does an RN provide therapeutic touch to a patient? Soft tissue is manipulated to improve lymphatic circulation and flow throughout the body. The fingers and hands apply light pressure on the meridians in the body to restore energy flow. The RN focuses on the inner self of the patient that connects mind, body, and spirit. The health care practitioner transfers healing energy to the patient using the hands, but without making contact.

1) Therapeutic touch is not classified as massage and bodywork, as there is no real contact. 2) Biological- and physiological-based therapies involve herbs, diet, and biofeedback. Therapeutic touch is not pharmacological, herbal, or based on clinical systems. 3) Therapeutic touch involves hands, but no actual contact, although the RN may tell the patient to assume a relaxed, inner focus. *4) Therapeutic touch is classified as an Energy Therapy.

Management of Patient Centered Care A patient who developed a venous thromboembolism following surgery dies on the third postoperative day from a pulmonary embolism. What is the appropriate description for this situation? A critical error A sentinel event A result of negligence A result of unsafe practice

1) There is no indication of error in this case. *2) A sentinel event is an incident where significant harm to patient has occurred. 3) Investigation of the event would be needed to determine if negligence contributed to the problem. 4) This complication is not a safety indicator.

Ethical, Legal, Regulatory, Professional Implications A nursing student asks an instructor what passing the RN licensure examination means. What is an accurate response? It provides the opportunity for an individual to practice in more than one state. It means that an individual demonstrates comprehensive nursing knowledge when the exam is passed. It demonstrates that an individual can practice within the legal and ethical guidelines in the profession. It means that an individual possesses minimal entry-level competence for safe and effective patient care

1) There is no way to guarantee ability. Passing a licensure examination can only ensure that an individual has met the minimum competencies in the nursing discipline. 2) Certification is the rating of credentials to indicate that an individual has achieved a level of ability higher than the minimal level of competence indicated by licensure. 3) There is no way to guarantee compliance with these guidelines. Passing a licensure examination can only ensure that an individual has met the minimum competencies in the nursing discipline. *4) A licensure examination assesses competency to ensure that an individual who passes the exam can function at the minimal level of competency and safety needed in the profession.

Management of Patient Centered Care What is the purpose of implementing the Roy Adaptation Model in clinical nursing practice? It supplies a dynamic process that identifies and meets the patient's health care goals. It identifies boundary disruption and helps patients find activities to restore stability. It offers a multistep process to assist the patient to adjust to and reach the highest level of functioning. It consists of activities that manipulate the environment and help patients achieve a balanced state of health.

1) This describes the King Model of Goal Attainment, not the Roy Adaptation Model. 2) This describes the Neuman Health-Care Systems Model, not the Roy Adaptation Model. *3) This describes the Roy Adaptation Model. 4) This describes the Johnson Behavioral System Model, not the Roy Adaptation Model.

Management of Patient Centered Care An RN administers an incorrect dose of a medication that the patient would have been able to obtain without a prescription. What is the appropriate action by the RN? Notify the pharmacist to send the correct medication. Initiate an incident report about the medication error. Tell the patient the correct dosage to use when continuing treatment with over-the-counter (OTC) medications. Document in the electronic health record that an incident report has been completed.

1) This does nothing to address the error the RN is accountable for. *2) An incident report should be initiated the person directly involved in the incident. 3) An incident report should be filed for any and all medication errors, regardless of classification of the drug. 4) Most authorities recommend that incident reports not be mentioned in the medical record.

Management of Patient Centered Care According to the American Nurses Association (ANA), what are the appropriate research competencies for the associate degree nursing (ADN) graduate? (Select all that apply.) Discusses the credibility of relevant research findings with colleagues. Assists in identifying problem areas in nursing practice. Assists with collection of data within an established, structured format. Demonstrates awareness of the value or relevance of research in nursing. Reads, interprets, and evaluates research for applicability to nursing practice.

1) This is a research competency of a BSN graduate. *2) This is a research competency of Associate degree nursing graduate. *3) This is a research competency of Associate degree nursing graduate. *4) This is a research competency of Associate degree nursing graduate. 5) This is a research competency of a BSN graduate.

Management of Patient Centered Care For what reason should an RN ask a novice RN about the novice RN's professional competency in urinary catheter insertion before delegating the task? To reinforce the authority of the RN To identify the need for supervision To encourage motivation to learn To prevent work overload

1) This is a safety issue, and reinforcing the authority of the RN is not necessary in this case. *2) Lack of competence and training is a safety issue, so the delegating RN needs to identify if the person performing the task has the knowledge and competency to perform the task. 3) This is a safety issue, and asking about competency in itself is unlikely to encourage motivation to learn. 4) This is a safety issue, and asking about competency will not prevent work overload.

Professional Role Development How does the National Council of State Boards of Nursing (NCSBN) update and revise licensure examination test plans? By participating in public policy issues By promoting and supporting research activities By performing comprehensive workplace analysis By providing economic and general welfare programs

1) This is an activity of the American Nurses Association (ANA). 2) This is an activity of the American Nurses Association (ANA). *3) The National Council of State Boards of Nursing (NCSBN) developed the new National Council Licensure Examination for Registered Nurses (NCLEX-RN) in 2010. These test plans are revised every 3 years, following a comprehensive workplace analysis, to ensure they are up-to-date with current practice. In conjunction with the revision of the test plan, the NCSBN also reviews and updates the passing score of the NCLEX. 4) This is an activity of the American Nurses Association (ANA).

Ethical, Legal, Regulatory, Professional Implications Which regulation uses restrictive language? The RN will provide teaching of health practices. The RN will administer the first dose of antibiotics. The LPN may discontinue a peripheral intravenous that is 3 inches or less in length. The LPN may perform the insertion of an indwelling urinary catheter using sterile technique.

1) This is an example of permissive regulation language, which allows practitioners to use judgment and make decisions to serve their purpose in performing their roles. 2) This is an example of permissive regulation language, which allows practitioners to use judgment and make decisions to serve their purpose in performing their roles. *3) Restrictive language restricts the practitioner to performing only the functions and procedures outlined in the regulation, in this case limiting the length of the peripheral intravenous. 4) This is an example of permissive regulation language, which allows practitioners to use judgment and make decisions to serve their purpose in performing their roles.

Professional Role Development A nursing student is reviewing the three learning domains in a particular component of the curriculum. Which would be an example of learning in the affective domain? Practicing wound care techniques Studying new content about a particular disease Calculating intravenous drop rates with various types of tubing Understanding the personal factors that influence patient's health

1) This is an example of the psychomotor domain that focuses on manipulative skills. 2) This is an example of the cognitive domain where knowledge is acquired. 3) See 1). *4) This is an example of the affective domain that involves values and attitudes.

Management of Patient Centered Care An RN is teaching a session on stress reduction to a group of patients. Which approach demonstrates democratic leadership by the RN? The RN establishes the goals and format for the group and encourages interaction to achieve results. The RN encourages each group member to determine their own goals and the method they will use to achieve them. The RN influences the choices the group makes among a number of choices available to them. The RN assists the members of the group in setting goals and processes for achieving success, for themselves.

1) This is authoritarian leadership style. 2) This is laissez-faire leadership style. 3) This is the motivational theory of leadership. *4) This is democratic leadership style.

Professional Role Development Which are examples of a collaborative activity? (Select all that apply.) The RN looks at an online tutorial. The RN invites a novice RN to go to journal club. The RN uses a computer with a bar coding system. The RN uses the Internet to find patient teaching handouts. The RN demonstrates how to use the Internet to research evidence-based practice.

1) This is not a collaborative effort. *2) By reaching out to a novice RN in an educational activity, the RN is collaborating in lifelong education. 3) This is not a collaborative effort. 4) This is not a collaborative effort. *5) By helping a colleague research evidence-based practice, the RN is collaborating in professional activity.

Ethical, Legal, Regulatory, Professional Implications A natural disaster has resulted in numerous patients requiring immediate care. When providing care to these patients, which action indicates an RN is applying the principle of justice? Give immediate care to patients who report they are injured the worst. Identify and treat patients based on personal standards of right and wrong. Provide for basic needs of all patients regardless of their injuries and social status. Treat patients first who have a preexisting diagnosis that could cause complications.

1) This is not an equal distribution of resources; it is focusing on patients who believe they need to be seen first. 2) This is bringing in personal biases into the provision of care. *3) This exemplifies the concept of justice: providing unbiased care, regardless of race, gender, social status, medical diagnosis, and economic level. 4) This is not an unbiased provision of care. The definition of justice is that the RN is taking preexisting conditions into consideration.

Management of Patient Centered Care Which action would RNs who are graduates of an associate degree nursing program perform to contribute to the evidence-based practice (EBP) research project being undertaken on their unit? Generating the research question Participating in data collection Presenting the project to the independent review board (IRB) Preparing the manuscript outlining the research findings

1) This is not at the RN level. *2) The RN can expect to participate in gathering data and participating on the unit. 3) This is not at the RN level. 4) This is not at the RN level.

Professional Role Development Which action by an RN demonstrates using the cognitive skill of analysis? Identifying a nursing diagnosis Using knowledge and strategies to support conclusions Predicting patient outcomes based on nursing actions Making informed decisions based on data not assumptions

1) This is the cognitive skill of interpretation. 2) This is the cognitive skill of explanation. 3) This is the cognitive skill of evaluation. *4) This is the cognitive skill of analysis: using information appropriately as a basis for decisions and actions.

*1) This is an appropriate task for a unlicensed assistive personnel (UAP) to perform. *2) This is an appropriate task for a unlicensed assistive personnel (UAP) to perform. 3) This task would be performed by a licensed practical nurse (LPN). 4) This task would be performed by a registered nurse (RN). 5) This task would be performed by a licensed practical nurse (LPN).

1) This is the definition of illness according to the King Model of Goal Attainment. 2) This is the definition of illness according to the Roy Adaptation Model. 3) This is the definition of illness according to the Orem Self-Care Model. *4) This is the definition of illness according to the Watson Model of Human Caring.

Professional Role Development An RN teaches a patient how to change a wound dressing, then allows the patient time to demonstrate redressing the wound. Which learning style is the RN using to help the patient learn? Visual Auditory Cognitive Kinesthetic

1) This is the visual learning style. 2) This is the auditory leaning style. 3) This is the cognitive learning style. *4) This is the kinesthetic learning style.

Ethical, Legal, Regulatory, Professional Implications Which situation would meet the exception requirements to obtaining an informed consent from the patient? A patient is upset and crying about a cancer diagnosis. A patient who speaks Russian has a medical interpreter. A patient received a prescribed intravenous (IV) beta blocker. A patient involved in a motor vehicle accident is unconscious and needs surgery.

1) This patient is still capable of being informed about care and requires an informed consent. 2) An interpreter needs to be obtained to relay the information for the health care provider when obtaining an informed consent, but consent must still be obtained. 3) Unless the medication has made the patient unconscious or incompetent, an informed consent for treatment is still required. *4) Patients in emergency situations who are unconscious meet the exception requirements when obtaining an informed consent for treatment.

Professional Role Development An RN uses direct and assertive speech to effectively communicate with nursing staff members during a patient crisis. Which Institute of Medicine (IOM) competency is the RN demonstrating? Informatics Quality improvement (QI) Interdisciplinary teamwork Evidence-based practice (EBP)

1) This situation is not an example of IOM's competency in informatics. 2) This situation is not an example of IOM's competency in quality improvement (QI). *3) This situation is an example of IOM's competency in interdisciplinary teamwork. 4) This situation is not an example of IOM's competencies in evidence-based practice (EBP).

Ethical, Legal, Regulatory, Professional Implications Which behavior by an RN would be appropriate to take in order to avoid a malpractice lawsuit? Maintain personal malpractice insurance. Communicate the minimal amount of patient information in report. Delay in investigating a patient's verbalize dissatisfaction with nursing care. Follow the American Nurses Association (ANA) Scope and Standards of Practice for Nurses.

1) This will do nothing to help avoid a malpractice suit but will be useful if involved in a suit. 2) Incomplete communication can contribute to a lawsuit. 3) Addressing concerns and working immediately to resolve problems promotes rapport and reduces the risk of lawsuits. *4) Following the Standards is the best way to ensure that care is provided consistently amongst professionals in the same clinical situation, and can avoid lawsuits.

Professional Role Development What does the decrease in mortality rates of the soldiers during the Crimean War by Florence Nightingale and her team of RNs demonstrate? Collective power of nursing Nursing education outcomes Patient-centered nursing care Practice of evidence-based nursing

1) This would be associated with professional nursing organizations. 2) Professional RN organizations developed after Nightingale. 3) This is an example of using statistics and drawing conclusions to change practice. *4) Nightingale's desire for educated/trained RNs led to her position in the Crimean War.

Ethical, Legal, Regulatory, Professional Implications What ethical principle does the RN violate when using social media to describe an incident with a patient that occurred at work? Veracity Advocacy Autonomy Confidentiality

1) Veracity is truth telling. Although the incident may have happened, it should not be told on social media. The act of using social media to talk about a patient significantly violates patient trust, and undermines the therapeutic relationship. 2) Advocacy is representing patient needs to a higher authority. 3) Patient autonomy means supporting patient self-determination. *4) The NCSBN Guide to professional boundaries notes this act as breach of patient confidentiality or privacy. The act of writing about a patient incident on social media, given the ubiquitous nature of the digital age, is a major infraction of a patient's fidelity, and breaches confidence. It should not be tolerated.

Management of Patient Centered Care When assigned to provide care for several patients, which patient would the RN assess first? A patient with nausea after eating A patient with a positive urine culture A patient with onset of confusion A patient with a temperature of 37.8LaTeX: ^\circC (100.2LaTeX: ^\circF)

1) While the RN will assess all the patients, the patient with nausea is not a priority. 2) While the RN will assess all the patients, the patient with a positive urine culture is not a priority. *3) A patient with a new onset of confusion indicates a change in mental status and this patient is the priority. A change in mental status could be indicative of a neurologic deficit, electrolyte imbalance, a hormonal imbalance, or a variety of other conditions. The RN needs to further assess the situation. 4) While the RN will assess all the patients, the patient with a temperature of 37.89°C (100.2°F) is not a priority.


Conjuntos de estudio relacionados

NMSP 10 Codes and Phonetic Alphabet

View Set

Women and Girls Initiative and Girls Summit 20th Year

View Set

CAWT120 Chapter 15 Seeking Employment Premium Quiz

View Set

AZ-303 - Module 4 - Implement VMs for Windows and Linux

View Set

Chapter 27 Antimicrobial Drugs and Drug Resistance

View Set

Articles of Confederation Vs Constitution

View Set